PDA

Archiv verlassen und diese Seite im Standarddesign anzeigen : Quantenspalt&Impuls


Eyk van Bommel
02.10.11, 16:46
Habe mir mal wieder zum DS-Experiment Gedanken gemacht.

A) Könnte man nicht am „1/2Durchlässigen Spiegel“ den Impuls messen den das abgelenkte Photon erzeugt?

O.K. komplizierter Aufbau aber….?

Gruß
EVB

EDIT: Ich wollte damit sagen: Es gibt eine Information die am Spiegel erzeugt wird, die es zumindest theoretisch erlaubt den Weg des Photons zu bestimmen nach dem das Photon den DS durchquert hat.

amc
04.10.11, 17:15
Könnte man nicht am „1/2Durchlässigen Spiegel“ den Impuls messen den das abgelenkte Photon erzeugt?
...
Ich wollte damit sagen: Es gibt eine Information die am Spiegel erzeugt wird, die es zumindest theoretisch erlaubt den Weg des Photons zu bestimmen nach dem das Photon den DS durchquert hat.

Hallo Eyk,

Du meinst also, warum wird bei einer Totalreflexion keine Welcher-Weg-Info geschaffen wird, die dann interferenzvernichtend ist? Solche Info wird nämlich offensichtlich nicht erzeugt, dass muss am Phänomen Totalreflexion liegen. Wenn man den Impuls misst, so verstehe ich es, müsste man auch ein wenig Energie entziehen. Bei einer Totalreflexion passiert dies wohl nicht.

Hier wurde schonmal fleißig darüber diskutiert, ist noch nicht abgeschlossen: http://www.quanten.de/forum/showthread.php5?p=62918#post62918


Freundlichst,
AMC

Eyk van Bommel
04.10.11, 17:55
Hallo amc,
Du meinst also, warum wird bei einer Totalreflexion keine Welcher-Weg-Info geschaffen wird, die dann interferenzvernichtend ist?

Mir geht es nicht direkt um die Totalreflexion sondern um die Ablenkung eines Photons am halbdurchlässigen Spiegel. Gut ist auch eine Art Totalreflexion..aber im 45° Winkel...

Wenn man den Impuls misst, so verstehe ich es, müsste man auch ein wenig Energie entziehen. Bei einer Totalreflexion passiert dies wohl nicht.

Bei der Totalreflexion wird sogar der doppelte Impulsübertragen. Einmal trifft das Photon auf und beim zweiten Mal wird es "abgestoßen". Richtig ist, dass das reflektierte oder hier das um 45° abgelenkte Photon etwas Rotverschoben wäre, wenn der Spiegel nicht den gesamten Impuls überträgt und selbst etwas abgelenkt wird.

Man könnte dem aber entgegenwirken? Man müsste „nur“ dem Spiegel schon vorher einen Impuls verpassen? Ihn z.B drehen lassen.

Man nimmt einen halbdurchlässigen Spiegel und lässt ihn sich drehen. Wenn der Spiegel stehen bleibt, dann hat das Photon den Weg x genommen – wenn nicht dann Weg A. Alternativ kann er sich auf die Photonenquelle zu bewegen- wenn er stehen bleibt…

Gut der Spiegel müsste sehr, sehr leicht sein – aber theoretisch….:rolleyes:



Gruß
EVB

Benjamin
04.10.11, 22:45
EDIT: Ich wollte damit sagen: Es gibt eine Information die am Spiegel erzeugt wird, die es zumindest theoretisch erlaubt den Weg des Photons zu bestimmen nach dem das Photon den DS durchquert hat.

Ich denke, die Impulsunschärfe des Spiegels lässt eine solche Messung nicht zu. Könntest du tatsächlich den Impuls des Spiegels so gut bestimmen, dass sich eine Photonreflexion messen ließe, wäre der Ort des Spiegels wiederum so unscharf, dass er das Interferenzmuster zerstört. Denn damit das Interferenzbild entsteht, muss reflektierte und nicht reflektierte Welle in fester Phasenbeziehung bleiben. Diese feste Phasenbeziehung wäre nicht gegeben, wenn der Ort der Reflexion (also der Spiegel) zu unscharf wäre.

Genau genommen handelt es sich sogar um viele Orte der Reflexion. Physikalisch wird das Photon nicht an der Oberfläche des Spiegels, oder gar an einem bestimmten Ort der Oberfläche reflektiert, sondern an allen möglichen Atomen des Spiegels. Die Überlagerung all dieser Wahrscheinlichkeiten führen erst zur resultierenden Welle wie wir sie beobachten. Der Spiegel als ganzes muss einer Impulsunschärfe unterliegen, die eine Impulsmessung unmöglich macht. Wären die Impulse der Spiegelatome nämlich genauer zu bestimmen, würden die Atome des Spiegel örtlich so unscharf sein, dass der Spiegel seine Eigenschaft verliert.

Ich bin mir eigentlich sicher, dass eine Impulsmessung eines Photons am Spiegel in der Art, dass man zwischen reflektiert und nicht reflektiert unterscheiden könnte, aus quantenmechanischen Gründen unmöglich ist.

amc
04.10.11, 23:38
Ich bin mir eigentlich sicher, dass eine Impulsmessung eines Photons am Spiegel in der Art, dass man zwischen reflektiert und nicht reflektiert unterscheiden könnte, aus quantenmechanischen Gründen unmöglich ist.

Du meinst, ohne dabei die Interferenz zu zerstören (oder meinst du grundsätzlich nicht?) - ja, ganz klar, das geht nicht. Das ist liegt im Wesen der Quantenwelt, die lässt sich nicht austricksen, aber Eyk kann natürlich daran tüfteln bis er grau wird, vielleicht klappts ja doch mal irgendwie ;)


Grüße,
AMC

richy
05.10.11, 01:04
Wenigstens theoretisch wird so ein Spiegelexperiment in Physikvorlesungen verwendet. Das Teilchen, besser Welle landet dann "auf selbem Weg" wieder in der Quelle. Auch der Zufall muesste dann mit zu der Welle gehoeren. Schon verrueckt.

amc
05.10.11, 01:26
Das Teilchen, besser Welle landet dann "auf selbem Weg" wieder in der Quelle.

Was meinst du genau mit dann? Dann, wenn man solch eine Impulsmessung durchführt und dadurch das Photon lokalisiert? Was ist damit gemeint, es landet zurück in der Quelle? Es wird doch am Spiegel lokalisiert? Wie kann man sich das vorstellen?

amc
05.10.11, 10:04
Das ist liegt im Wesen der Quantenwelt, die lässt sich nicht austricksen

Hallo Eyk,

ist das denn der Grund deiner Überlegung - ob man vielleicht doch eine Weg-Info erhalten kann und sich gleichzeitig die Interferenz erhalten lässt? Oder worum geht es dir?


Freundlichst,
AMC

Eyk van Bommel
05.10.11, 19:18
Hallo amc,
ist das denn der Grund deiner Überlegung - ob man vielleicht doch eine Weg-Info erhalten kann und sich gleichzeitig die Interferenz erhalten lässt? Oder worum geht es dir?
Ja darum ging es "natürlich":o
wäre der Ort des Spiegels wiederum so unscharf, dass er das Interferenzmuster zerstört.
Dass die Impulsessung eines Objektes durch ein Photon, automatisch eine Ortsunschärfe bedingt war mir nicht klar. Die Pioneer-Sonde ließ sich ja ziemlich gut "orten"und wurde durch Photonen abgebremst. :D ;).

Gruß
EVB

PS: Noch ne Überlegung. Angenommen der Weg des Photons wird durch ein Gas gemessen. Wir setzen also in einen der beiden Wege eine Kiste, angefüllt mit Molekülen die Licht empfindlich sind.

Trift das Photon auf ein solches Molekül, dann gilt dies als Messung.

Jetzt wird die Konzentration langsam so eingestellt, dass A) 10% B) 35% C) 50% D) 75% der Photonen die den Weg einschlagen auch wechselwirken werden.

Ab wann wird die Interferenz messbar gestört.

Das komisch ist ja, nimmt das Photon den anderen Weg, dann muss es sozusagen wissen, dass es auf dem anderen Weg gemessen worden wäre.

Andererseits, selbst bei 75% Wahrscheinlichkeit, werden wir nicht mit Sicherheit sagen können, welcher weg es eingeschlagen wurde? Keine „welcher Weg“ Information???

Die Idee kam mir als ich einen Aufbau gesehen hatte, indem der Laserstrahl auf dem gesamten Weg durch Nebel sichtbar gemacht wurde. Die Interferenz wurde nicht (sichtbar) gestört. Dabei hätte ja das ein oder andere Photon, hätte es den anderen weg genommen, gestreut/=gemessen werden können.

Gruß
EVB

EDIT: Ich merke gerade das ist imho Vergleichbar zu einem Experiment mit einem Polarisationsfilter :(

JoAx
05.10.11, 20:18
Hallo Eyk!


Angenommen der Weg des Photons wird durch ein Gas gemessen.

Oder man nimmt Elektrone und eine Nebelkammer. Die Frage ist nun, ob man aus der (dicken) Spur ableiten kann, durch welchen der beiden (sehr eng beieinander stehenden) Spalten das Elektron gegangen ist. Das dürfte imho schwierig werden.


Jetzt wird die Konzentration langsam so eingestellt, dass A) 10% B) 35% C) 50% D) 75% der Photonen die den Weg einschlagen auch wechselwirken werden.

Ab wann wird die Interferenz messbar gestört.


Das resultierende Bild dürfte eine Mischung aus Interferenz und keine Interferenz werden. Was überwiegt, hängt dann von den Einstellungen ab.


Gruß, Johann

amc
05.10.11, 20:30
Jetzt wird die Konzentration langsam so eingestellt, dass A) 10% B) 35% C) 50% D) 75% der Photonen die den Weg einschlagen auch wechselwirken werden.

Ab wann wird die Interferenz messbar gestört.

Ich denke, wenn manche Photonen wechselwirken und manche nicht, dann erhält man auch immer beides, Interferenzmuster und kein Interferenzmuster. Die Frage ist natürlich welches Muster ist ausgeprägter, das hängt dann sicher von dem Verhältnis der wechselwikungsfreien zu nicht ww-freien Photonen ab. Wenn z.B. immer nur 1% ww-frei ist, dann müsste man wohl entsprechend länger Photonen schießen, damit das Interferenzbild klar von den statistischen Ausreißern der wechselgewirkten Photonen ( oder gewechselwirkten :rolleyes: ;) ) zu unterscheiden ist.

Oh, ich sehe Johann hats auch schon so erwähnt ...

Freundlichst,
AMC

richy
05.10.11, 21:15
Hi amc
Was meinst du genau mit dann?Das bezog sich auf eine ganz einfache Totalreflexion beim Einfachspalt.

Quelle ------- Spalt ---------Spiegel

Geringe Intensitaet. Bei einem Schirm statt Detektor ist Auftrittsort daher zufaellig.
Angenommen: Keine Dekohaerenz.
Ich meine der Spiegel war sogar plan.
Was passiert ?

Benjamin
05.10.11, 22:37
Du meinst, ohne dabei die Interferenz zu zerstören (oder meinst du grundsätzlich nicht?) - ja, ganz klar, das geht nicht. Das ist liegt im Wesen der Quantenwelt, die lässt sich nicht austricksen, aber Eyk kann natürlich daran tüfteln bis er grau wird, vielleicht klappts ja doch mal irgendwie ;)

Die meisten modernen Interferometer arbeiten mit halbdurchlässigen Spiegeln. Auch das Michelson-Interferometer funktioniert mit solchen. Es entsteht sozusagen ein Interferenzbild aus durchgelassenen und reflektierten Strahl. Das ist eine sehr elegante und vielfach nützliche Art Interferenzmuster zu erzeugen.

Man könnte der Versuchung unterliegen, so ein Interferometer dafür zu nutzen, einzelne Photonen durchzuschicken und ihren Weg bestimmen zu wollen. Wichtig dabei ist, dass das Interferenzmuster nur dann entsteht, wenn keine Information darüber vorliegt, welchen Weg das Photon nimmt, sprich, ob es am halbdurchlässigen Spiegel reflektiert oder durchgelassen wird.
Eine Wegbestimmung könnte zB damit gelingen, dass man eine Impulsmessung am Spiegel durchführt. Ein reflektiertes Photon überträgt nämlich einen Impuls auf den Spiegel, wobei ein durchgelassenes Photon keinen Impuls überträgt. Imho könnte man denken, es wäre möglich den Weg mittels Impulsmessung am Spiegel zu bestimmen und auch ein Interferenzmuster zu bekommen.
Was spricht gegen solch eine Impulsmessung?

Nun, ich denke, es ist am Spiegel an sich nicht möglich den Impuls so genau zu messen, dass man auf den Weg des Photons rückschließen könnte. Und zwar muss es an den physikalischen Eigenschaften des Spiegels selbst liegen. Seine physikalische Eigenschaft mit Licht in der Art in Wechselwirkung zu treten, dass die Phase der Welle kohärent bleibt, muss mit einer so großen Impulsunschärfe des Spiegels verbunden sein, dass auf den Weg des Photons nicht rückgeschlossen werden kann.
Warum es zu dieser Impulsunschärfe kommt, habe ich im vorherigen Beitrag erläutert.

Eyk van Bommel
06.10.11, 07:38
Die Frage (mein letztes Beispiel) die mir durch den Kopf ging war:
Wenn der Weg A in 5% der Fälle zu einer Wechselwirkung führt, wobei die Bewegung der (Mess-)Moleküle als nicht determiniert zu verstehen ist (anders wie beim Polarisationsfilter!:rolleyes: ) dann kommen insgesamt nur 97,5% der Photonen am DS an.

95% der Photonen sollten eine Interferenz aufweisen. 2,5% sind durch A „vernichtet“ (kommen also nie an!) und die restlichen 2,5% sind den Weg B gegangen, wären aber bei A gemessen worden = keine Interfenz?
@Benjamin
Seine physikalische Eigenschaft mit Licht in der Art in Wechselwirkung zu treten, dass die Phase der Welle kohärent bleibt, muss mit einer so großen Impulsunschärfe des Spiegels verbunden sein, dass auf den Weg des Photons nicht rückgeschlossen werden kann.
Finde diese Antwort ja gut, aber…
Also entweder ist der Spiegel so leicht, dass er selbst quantenphysikalischen Gesetzten unterliegt (unschärfe) oder so schwer, dass eine Bewegungsänderung nicht mehr gemessen werden kann?

Gruß
EVB

Benjamin
06.10.11, 17:58
Die Frage (mein letztes Beispiel) die mir durch den Kopf ging war:
Wenn der Weg A in 5% der Fälle zu einer Wechselwirkung führt, wobei die Bewegung der (Mess-)Moleküle als nicht determiniert zu verstehen ist (anders wie beim Polarisationsfilter!:rolleyes: ) dann kommen insgesamt nur 97,5% der Photonen am DS an.

95% der Photonen sollten eine Interferenz aufweisen. 2,5% sind durch A „vernichtet“ (kommen also nie an!) und die restlichen 2,5% sind den Weg B gegangen, wären aber bei A gemessen worden = keine Interfenz?


Hm? "die restlichen 2,5% sind den Weg B gegangen, wären aber bei A gemessen worden " ... Wenn sie in A gemessen werden, sind sie den Weg A gegangen und nicht den Weg B!

Ab wann wird die Interferenz messbar gestört.

Was meinst du mit "die Interferenz wird messbar gestört"? Wie es in keinem Experiment 100%ig dem Modell entsprechende Ergebnisse gibt, so gibt es auch kein perfektes Interferenzbild, weil wir uns Modellen bedienen, die die Natur vereinfacht darstellen. Somit musst du "messbare Störung" genauer definieren.

Das komisch ist ja, nimmt das Photon den anderen Weg, dann muss es sozusagen wissen, dass es auf dem anderen Weg gemessen worden wäre.

Ja, richtig. Mache aber nicht den Fehler, dir vorzustellen, da gäbe es ein Teilchen, das sich entweder auf dem Weg A oder auf dem Weg B befindet. Das gibt es nämlich nicht! Die korrekte Beschreibung der Lichtausbreitung gelingt über eine Wellenfunktion. Genauer heißt das: Eine Angabe über Ort und Zeit einer elektromagnetischen Wechselwirkung gehorcht einer Wellengleichung, und diese ist für Weg A und B definiert. Das wirklich Außergewöhnliche liegt nun in dem Umstand, dass wir aus dieser Wellengleichung nur erfahren mit welcher Wahrscheinlichkeit eine solche Wechselwirkung eintritt. Warum das so ist, weiß kein Mensch auf dieser Welt, oder zumindest konnte es noch niemand der breiten Öffentlichkeit verständlich machen.
Das sogenannte Photon, dem du versuchst einen konkreten Ort zu einer konkreten Zeit zuzuschreiben, existiert zu keiner Zeit an einem solchen Ort, außer zum Zeitpunkt seiner Messung. Davor jedoch existiert es als eine Art Welle und diese Welle breitet sich auf Weg A und B gleichzeitig aus.
Das ist nicht logisch und widerspricht dem menschlichen Verstand ganz allgemein, aber so sieht das Bild aus, das wir uns von der Natur gemacht haben. Momentan scheint es noch immer das anschaulichste zu sein.

Also entweder ist der Spiegel so leicht, dass er selbst quantenphysikalischen Gesetzten unterliegt (unschärfe) oder so schwer, dass eine Bewegungsänderung nicht mehr gemessen werden kann?

Jedes physikalische Objekt gehorcht qm Gesetzen. Die Frage ist nur, ob diese Gesetze für unsere Anschauung dienlich sind. Will man den Impulsübertrag eines Fußballes auf einen Spiegel beschreiben, braucht es dieser Gesetze nicht. Willst du aber den Impulsübertrag eines Photons auf diesen Spiegel untersuchen, musst du dich derer sehr wohl bedienen. Der Spiegel ist nicht unbedingt zu schwer, wenngleich er auch aus 10^23 Atomen bestehen mag, er ist vor allem zu unruhig. Die Fluktuationen des Spiegels selbst übertreffen jede mögliche Messung des Photonenimpulses, und diese Fluktuationen sind nichts anderes als die Impulsunschärfe des Spiegels.

Benjamin
06.10.11, 18:06
Das sogenannte Photon, dem du versuchst einen konkreten Ort zu einer konkreten Zeit zuzuschreiben, existiert zu keiner Zeit an einem solchen Ort, außer zum Zeitpunkt seiner Messung. Davor jedoch existiert es als eine Art Welle und diese Welle breitet sich auf Weg A und B gleichzeitig aus.

Ach ja, was noch wesentlich ist: Um die Wellenfunktion des Photons zu verändern, reicht allein die Möglichkeit einer Wechselwirkung, sprich Messung aus. Eine wirkliche Messung zerstört die Wellenfunktion, und die Energie des Photons geht in eine andere Energieforum über. Aber allein dadurch, dass du zB den Weg A mit Gas flutest, veränderst du die Wellenfuntkion, weil dadurch eine Wechselwirkung möglich wird. Sie muss gar nicht geschehen, nur die Möglichkeit alleine genügt!

JoAx
06.10.11, 18:26
Hallo Benjamin!

Ach ja, was noch wesentlich ist: Um die Wellenfunktion des Photons zu verändern, reicht allein die Möglichkeit einer Wechselwirkung, sprich Messung aus. Eine wirkliche Messung zerstört die Wellenfunktion, und die Energie des Photons geht in eine andere Energieforum über. Aber allein dadurch, dass du zB den Weg A mit Gas flutest, veränderst du die Wellenfuntkion, weil dadurch eine Wechselwirkung möglich wird. Sie muss gar nicht geschehen, nur die Möglichkeit alleine genügt!

Das ist aber definitiv nicht richtig. Die Möglichkeit an sich zerstört noch nicht die Interferenz. Geht ein Photon durch, ohne auch nur ein einziges Molekül angeregt zu haben, dann wird es auch interferieren. Warum sollte es das nicht tun können?


Gruß, Johann

Benjamin
06.10.11, 19:53
Das ist aber definitiv nicht richtig. Die Möglichkeit an sich zerstört noch nicht die Interferenz. Geht ein Photon durch, ohne auch nur ein einziges Molekül angeregt zu haben, dann wird es auch interferieren. Warum sollte es das nicht tun können?


Achso? :)
Du meinst, Licht kann ein Gas durchlaufen, ohne ein Molekül anzuregen? Heißt das dann, das Licht läuft durchs Gas mit Vakuumlichtgeschwindigkeit?
Das tut es gewiss nicht! Ein Gas, wie die Luft, ist ein optisches Medium mit einer Brechzahl, daher ist Licht auch langsamer darin.
Wenn du sagst, Photonen können durchs Gas laufen, ohne mit dem Gas in Wechselwirkung zu treten - und ich denke, das meintest du doch mit "Molekül anregen" - dann muss das Licht auch mit Vakuumlichtgeschwindigkeit durch das Gas laufen, denn warum sollte es langsamer werden?

Kannst du mir das erklären?

Eyk van Bommel
06.10.11, 21:26
Du meinst, Licht kann ein Gas durchlaufen, ohne ein Molekül anzuregen? Heißt das dann, das Licht läuft durchs Gas mit Vakuumlichtgeschwindigkeit?

Müssen DS-Experimente bzw. am Interferometer im Vakuum durchgeführt werden:confused:

JoAx sprach ja von Anregung, die findet nicht automatisch statt wenn das Licht durch ein Gas/Luft „stiefelt“?

Hm? "die restlichen 2,5% sind den Weg B gegangen, wären aber bei A gemessen worden " ... Wenn sie in A gemessen werden, sind sie den Weg A gegangen und nicht den Weg B!

Ich habe mich wohl nicht klar ausgedrückt.

Bei A befindet sich ein Gas (oder Plasma) in einer Konzentration, dass dieses mit einer Wahrscheinlichkeit von 5% das abkommende Photon absorbiert/= detektiert (Molekül wird gespalten. Spaltprodukt könnte gemessen werden…oder so).

Dies entspricht einem (eingeschalteten) Detektor auf dem Weg A – aber eben nur in 5% der Fälle.

Somit kommen auf dem Weg A nur 95% der Photonen über den Weg A „hinten am Schirm an“ = 100% Interferenz. Richtig? Da auf dem Weg B nicht gemessen wird auch = 100% Interferenz.

Schlagen die Photonen den Weg B ein, dann kommen 100% an. ABER 5% der Photonen würden „spüren“, dass auf dem Weg A ein Detektor „eingeschaltet“ ist (ein Molekül im Weg gewesen WÄRE(!) Sie kommen zwar am Schirm an zeigen daher aber keine Interferenz?

Das bedeutet 5% der Photonen zeigt keine Interferenz, da sie den Weg B gegangen sind. Aber 100% der Photonen über den Weg A zeigen Interferenz.

Da keiner sagen kann, welches Photon absorbiert wird, kann kein Beobachter herausfinden, welchen Weg das Photon genommen hat, wenn es hinten ankommt. Warum sollten dann nicht 100% der Photonen die den Weg B einschlagen eine Interferenz aufweisen?

Es geht also auch ein Stück um die Rolle des Beobachters?

Gruß
EVB

amc
06.10.11, 21:31
Hallo Benjamin, Hallo Johann,

ich glaube ihr redet etwas aneinander vorbei. Benjamin sagte nicht, dass eine mögliche Wechselwirkung die Interferenz zerstört, sondern die Wellenfunktion verändert sich dadurch. Und Johann sagte nicht, dass Licht grundsätzlich nicht wechselwirkt, wenn es durch ein Gas läuft, das bezog sich lediglich auf einzelne Photonen.

Grüße

Benjamin
06.10.11, 21:32
JoAx sprach ja von Anregung, die findet nicht automatisch statt wenn das Licht durch ein Gas/Luft „stiefelt“?

Was meinst du mit Anregung? Wenn ein Photon ein Molekül oder Atom oder was auch immer anregt, ist es verschwunden! Ein Photon ist die kleinste Anregung des elektromagnetischen Feldes. Ein Photon kann sich weder teilen, noch kann es beim Vorbeilaufen Materie anregen, da es ja die kleinste Anregung ist, und wenn das Photon Materie anregt, wird es dabei unweigerlich zerstört.

Eyk van Bommel
06.10.11, 21:38
Dann gehst du davon aus, dass das Licht wenn es durch das Glas wandert, ständig vernichtet und neu gebildet wird:confused:

amc
06.10.11, 21:49
Ein Photon ist die kleinste Anregung des elektromagnetischen Feldes. Ein Photon kann sich weder teilen, noch kann es beim Vorbeilaufen Materie anregen...

Die Energie eines Photons ist je bekanntlich, abhängig von der Frequenz, ein vielfaches von h - es kann also, wie du sagst, nicht einen Teil der Energie abgeben, und dann rotverschoben weiterfliegen? Ich weiß es nicht, frage nur ...

Benjamin
06.10.11, 21:58
Dann gehst du davon aus, dass das Licht wenn es durch das Glas wandert, ständig vernichtet und neu gebildet wird:confused:

Ja, das kommt der Sache am nähesten! Wobei dieses ständige Vernichten und Erzeugen nicht beobachtet werden kann. Entscheidend ist, dass die Frequenz des Photons vor dem Eintritt in das Glas und nach dem Austritt aus dem Glas dieselbe ist. Das beobachtet man auch! Und es bedeutet, dass das Photon dem Glas keine Energie übertragen hat. Die Gesamtenergie des Glases ist vor und nach dem Durchlaufen des Photons die gleiche.

Wie gesagt, die Möglichkeiten wie ein Photon sich verhalten kann, bestimmen das beobachtete Verhalten desselben. Beim DS ist es so, dass nur die Möglichkeit, dass das Photon durch einen zweiten Spalt laufen könnte, das Gesamtergebnis verändert. Erst wenn du die Möglichkeit, dass das Photon durch einen zweiten Spalt laufen könnte, völlig ausschließt, wirst du kein Interferenzmuster mehr sehen. Je größer diese Wahrscheinlichkeit wird (bis 50%), umso schöner wird das Interferenzmuster sichtbar.

Eyk van Bommel
06.10.11, 22:10
Entscheidend ist, dass die Frequenz des Photons vor dem Eintritt in das Glas und nach dem Austritt aus dem Glas dieselbe ist. Das beobachtet man auch! Und es bedeutet, dass das Photon dem Glas keine Energie übertragen hat. Die Gesamtenergie des Glases ist vor und nach dem Durchlaufen des Photons die gleiche.
Das ist klar. Nur halte ich es für fraglich, dass die Beschreibung „statthaft“ ist. Nach einer Anregung ist für mich die Richtung der Emission des Photons nicht vorhersagbar. Dies ist in einem durchlässigen Medium aber nicht der Fall?

Ich ging davon aus, dass das Licht in einem durchlässigen Medium durch das EM-Feld nur abgelenkt wird ohne Impulsübertrag (im Sinne eines elastischen Stoßes zweier gleichschwerer-/Träger Teilchen).

Hier von Anregung zu sprechen halte ich für falsch:o Dies liegt womöglich an meinem chemisch-physiklaischen Hintergrund:o

Gruß
EVB

Benjamin
06.10.11, 22:33
Ich ging davon aus, dass das Licht in einem durchlässigen Medium durch das EM-Feld nur abgelenkt wird ohne Impulsübertrag (im Sinne eines elastischen Stoßes zweier gleichschwerer-/Träger Teilchen).


Zunächst einmal wird Licht von einem em Feld nicht beeinflusst. Es sind die Elektronen der Materie, die mit Licht in Wechselwirkung treten. Hier von einem klassischen Stoß zu sprechen, wäre zu sehr vereinfacht, wenngleich es auch solche Analogieschlüsse gibt, wo Photon-Elektron-WW wie ein relativistischer Stoßprozess von Teilchen beschrieben wird.
Aber auch so eine Beschreibung trifft die Natur nicht exakt.

Nach einer Anregung ist für mich die Richtung der Emission des Photons nicht vorhersagbar. Dies ist in einem durchlässigen Medium aber nicht der Fall?

Die Emission eines einzelnen Photons ist zu tiefst unvorhersehbar, vor allem die Richtung desselben. Wie schon gesagt, Photonen bewegen sich nicht wie klassische Teilchen auf einer geraden Bahn durch den Raum. Sie bewegen sich noch nicht einmal auf einer krummen oder irgendeiner beliebigen Bahn, sie bewegen sich auf gar keiner Bahn. Sie breiten sich wie Wellen aus. Das ist die exakteste Vorstellung über die Bewegung von Licht, die wir haben. Auch im Glas ist es nicht anders. Dh. auch dort bewegt sich Licht quasi in alle Richtungen, und erst aus den Überlagerungen aller Möglichkeiten (Streuungen mit eingeschlossen), die im Glas geschehen können, ergibt sich dann die resultierende Welle, wie wir sie klassisch beobachten, mit Ausbreitungsrichtung, Reflexions- und Absorptionskoeffizient, usw. Sogar die Reflexion an einem Spiegel mit dem berühmten Gesetz, dass Einfallswinkel gleich Ausfallswinkel ist, ist eine Folge von allen möglichen Überlagerungen. Klassisch nennt man das das Huygensschen Prinzip. Es gilt im Grunde auch in der QFT, jedoch noch weiter verallgemeinert und vor allem quantisiert!

Eyk van Bommel
06.10.11, 22:36
Mache aber nicht den Fehler, dir vorzustellen, da gäbe es ein Teilchen, das sich entweder auf dem Weg A oder auf dem Weg B befindet. Das gibt es nämlich nicht! Die korrekte Beschreibung der Lichtausbreitung gelingt über eine Wellenfunktion. Genauer heißt das: Eine Angabe über Ort und Zeit einer elektromagnetischen Wechselwirkung gehorcht einer Wellengleichung, und diese ist für Weg A und B definiert.
Ich hätte ja kein Problem damit, aber...
Das wirklich Außergewöhnliche liegt nun in dem Umstand, dass wir aus dieser Wellengleichung nur erfahren mit welcher Wahrscheinlichkeit eine solche Wechselwirkung eintritt.
Mein Problem ist ja, dass das Photon nur „spürt“ dass seine Anwesenheit detektiert worden wäre (hätte es nur den anderen Weg eingeschlagen). Es findet aber keine (messbare) Wechselwirkung auf dem „anderen Weg“ statt ("mit der zweitenhälfte der Welle")

Das Photon trifft am Schirm mit voller Intensität auf. Was hat sich geändert und wie?? Welche Information lässt das Photon lokalisieren? Und wie schnell? Erst am theoretischen „Treffpunkt?“

Mir hat es den Anschein als wären die beiden Wellen (Aufenthaltswahrscheinlichkeiten) verschränkt?

Gruß
EVB

Benjamin
06.10.11, 22:48
Mein Problem ist ja, dass das Photon nur „spürt“ dass seine Anwesenheit detektiert worden wäre (hätte es nur den anderen Weg eingeschlagen). Es findet aber keine (messbare) Wechselwirkung auf dem „anderen Weg“ statt ("mit der zweitenhälfte der Welle")

Das Photon trifft am Schirm mit voller Intensität auf. Was hat sich geändert und wie?? Welche Information lässt das Photon lokalisieren? Und wie schnell? Erst am theoretischen „Treffpunkt?“

Pass auf! Ich geb dir ein Modell, das all diese Fragen erklären kann:
Stell dir vor, Licht besteht aus einem Teilchen, dem Photon, und einer Welle, auf der es sozusagen schwimmt. Das Photon kann nur dort sein, wo die Welle ist, und die Welle breitet sich ganz normal, wie eine em Welle aus. Das heißt, sie wird gestreut, gebeugt und interferiert. Sie kann aber Materie nicht beeinflussen, das kann nur das Photon, und zwar immer mit voller Energie hf.

Ich gebe zu, dieses Modell habe ich geklaut, es stammt von Louis de Broglie und David Bohm. Und es kann den DS so erklären, wie du es gern hättest. http://de.wikipedia.org/wiki/Bohmsche_Mechanik

JoAx
06.10.11, 23:07
...
Kannst du mir das erklären?


Ich kann's ja versuchen, Benjamin. (Wie ich das sehe, zumindest :) )

Ein Gedankenexperiment mit einer Art Michelson-Morley-Interferometer, also.
1. Beide Röhre sind evakuiert -> Interferenz

2. Ich lasse in eine der Röhren 3 Moleküle, die, falls es zu einer WW mit dem Lichtquant kommt, mir das "verraten" würden.
Ist da die bloße Möglichkeit einer WW gegeben? - Ja.
Ist die Interferenz in jedem Fall zerstört? - Nein. Nur wenn die WW tatsächlich stattgefunden hat. (Vlt. auch dann, wenn die WW tatsächlich hätte stattfinden können/sollen? Ist eine Frage.)

3. Eine der Röhren wird geflutet. (Dein Fall.)
Ist da die bloße Möglichkeit einer WW gegeben? - Nein. D.h. - wenn das Photon durch die geflutete Röhre geht, dann ist die Wahrscheinlichkeit einer WW praktisch 100%. Das würde ich nicht mit - Möglichkeit zur WW - bezeichnen. "Sehe" ich nichts, bedeutet es definitiv, dass das andere Rohr "genommen wurde".
Ist die Interferenz in jedem Fall zerstört? - Ja.
Hier ist immer die Möglichkeit gegeben an Welcher-Weg-Information zu kommen. Selbst dann, wenn man es gar nicht möchte. "Das zerstört die Interferenz". (Auch dann, wenn man nicht hin sieht. :D )


So, meine ich, wäre es korrekt "beschrieben". imho


Gruß, Johann

JoAx
06.10.11, 23:16
Ich habe mich wohl nicht klar ausgedrückt.
...

Dadurch ist es aber auch nicht klarer geworden, Eyk.
Kannst du dein Gedankenexperiment, sein Aufbau bitte so einfach wie möglich aber exakt beschreiben? Eine Zeichnung würde auch nicht schaden.


Gruß, Johann

JoAx
06.10.11, 23:19
Benjamin sagte nicht, dass eine mögliche Wechselwirkung die Interferenz zerstört,


Ich weiß nicht, amc. Das:


weil dadurch eine Wechselwirkung möglich wird. Sie muss gar nicht geschehen, nur die Möglichkeit alleine genügt!

hört sich doch nach bloßer Möglichkeit einer WW an. Oder nicht?


Gruß, Johann

MCD
06.10.11, 23:20
Das beobachtet man auch! Und es bedeutet, dass das Photon dem Glas keine Energie übertragen hat. Die Gesamtenergie des Glases ist vor und nach dem Durchlaufen des Photons die gleiche.

Hi Benjamin,

wie erklärst du die Erwärmung des Mediums (hier Glas) infolge Photoneneinwirkung, wenn nicht mit Energieübertragung?:confused:

Gr.
MCD

JoAx
07.10.11, 00:25
Sie kann aber Materie nicht beeinflussen, das kann nur das Photon, und zwar immer mit voller Energie hf.


So toll es auch klingen mag. Wieso kann diese Welle "ihr" Photon/"ihre" Materie (Elektron, für die gilt dann ja das gleiche) beeinflussen, aber nicht die "fremde" Materie?
Und das ist nicht die einzige offene Frage.


Gruß, Johann

amc
07.10.11, 08:17
Ich weiß nicht, amc. Das:
hört sich doch nach bloßer Möglichkeit einer WW an. Oder nicht?
Gruß, Johann

Ja das stimmt Johann, Benjamin sprach von einer lediglich möglichen WW. Jedoch nicht, das dadurch die Interferenz zerstört wird, sondern dadurch verändert sich die Wellenfunktion, die beschreibt, mit welcher Wahrscheinlichkeit das Photon wann und wo wechselwirken wird (ist das richtig?). Wenn eine Wellenfunktion die Wechselwirkungswahrscheinlichkeiten des Quantenobjektes beschreibt, dann ist natürlich klar, dass eine mögliche WW diese Wellenfunktion maßgeblich verändert. Und dies geschieht, so wie ich es verstehe, nichtlokal und instantan.

Grüße, AMC

amc
07.10.11, 08:24
Hi Benjamin,

wie erklärst du die Erwärmung des Mediums (hier Glas) infolge Photoneneinwirkung, wenn nicht mit Energieübertragung?:confused:

Gr.
MCD

Ich will mal versuchen vorzugreifen. Gibt ja eigentlich nur zwei Möglichkeiten: Entweder wird den Photonen Energie entzogen (möglichwerweise durch Absorption und Reemission eines Photons mit niedriger Energie), oder einige bleiben im Glas stecken (werden nach Absorption nicht wieder reemittiert). Benjamin, was ist richtig?

Grüße, AMC

Benjamin
07.10.11, 12:23
2. Ich lasse in eine der Röhren 3 Moleküle, die, falls es zu einer WW mit dem Lichtquant kommt, mir das "verraten" würden.
Ist da die bloße Möglichkeit einer WW gegeben? - Ja.
Ist die Interferenz in jedem Fall zerstört? - Nein. Nur wenn die WW tatsächlich stattgefunden hat. (Vlt. auch dann, wenn die WW tatsächlich hätte stattfinden können/sollen? Ist eine Frage.)

3. Eine der Röhren wird geflutet. (Dein Fall.)
Ist da die bloße Möglichkeit einer WW gegeben? - Nein. D.h. - wenn das Photon durch die geflutete Röhre geht, dann ist die Wahrscheinlichkeit einer WW praktisch 100%. Das würde ich nicht mit - Möglichkeit zur WW - bezeichnen. "Sehe" ich nichts, bedeutet es definitiv, dass das andere Rohr "genommen wurde".
Ist die Interferenz in jedem Fall zerstört? - Ja.
Hier ist immer die Möglichkeit gegeben an Welcher-Weg-Information zu kommen. Selbst dann, wenn man es gar nicht möchte. "Das zerstört die Interferenz". (Auch dann, wenn man nicht hin sieht. :D )



Ich denke, es wäre zunächst einmal wichtig "Wechselwirkung" zu definieren. Ich verstehe unter WW einen physikalischen Vorgang, wo Energieaustausch bzw. -umwandlung stattfindet. Das ist auch so ziemlich die gängigste Definition in der Physik.

Demnach durchlaufen Photonen definitiv Glas oder Gas, ohne mit ihm in WW zu treten. Sonst könnte man niemals einzelne Photonen durch Polarisationsfilter und dergleichen schicken, um sie nachher interferieren zu lassen. Photonen können durch Materie präpariert werden, ohne dass sie dabei Energie abgeben!

Ich zeig dir an einem einfachen Beispiel, dass deine Überlegungen falsch sind: Du kannst Photonen sowohl durch Luft als auch durch Glas schicken und erhältst trotzdem einwandfreie Interferenz. Du kannst sogar einen Strahl durch Gas schicken und den anderen durch Vakuum und du erhältst trotzdem Interferenz! Die meisten Interferenzversuche werden in Luft durchgeführt, oder glaubst du Michelson und Morley haben für ihre Versuche ein Ultrahochvakuum hergestellt?
Somit siehst du, Photonen können (einzeln!) durch Materie laufen und ergeben trotzdem ein Interferenzbild, und das obwohl sie dabei ihre Laufzeit verändern! Dh. die Lichtgeschwindigkeit ist definitiv geringer in der Materie, und dennoch wird das Interferenzbild beim Durchlaufen derselben nicht zerstört.

Warum? Weil nur die Möglichkeit einer WW - also die Möglichkeit von Energieübertragung auf die Materie - die Bewegung des Photons beeinflussen.

Benjamin
07.10.11, 12:29
Hi Benjamin,

wie erklärst du die Erwärmung des Mediums (hier Glas) infolge Photoneneinwirkung, wenn nicht mit Energieübertragung?:confused:


Ich will mal versuchen vorzugreifen. Gibt ja eigentlich nur zwei Möglichkeiten: Entweder wird den Photonen Energie entzogen (möglichwerweise durch Absorption und Reemission eines Photons mit niedriger Energie), oder einige bleiben im Glas stecken (werden nach Absorption nicht wieder reemittiert). Benjamin, was ist richtig?


Ja, das ist in der Tat die Erklärung. Die Möglichkeit einer WW bedeutet natürlich, dass Photonen auch Energie auf das Glas übertragen können. Und sie tun es auch, manche von ihnen zumindest, andere laufen durchs Glas durch, wieder andere werden reflektiert. Dass Glas oder jedes andere Material beim Durchlaufen von Licht erwärmt wird, liegt daran, dass eben manche Photonen mit der Materie wechselwirken und somit Energie zuführen. Es muss auch so sein, besteht immerhin die Möglichkeit der WW. Und bei genügend großer Anzahl an Photonen lässt sich auch beobachten, dass manche von ihnen von dieser Möglichkeit Gebrauch machen.

Benjamin
07.10.11, 12:39
So toll es auch klingen mag. Wieso kann diese Welle "ihr" Photon/"ihre" Materie (Elektron, für die gilt dann ja das gleiche) beeinflussen, aber nicht die "fremde" Materie?
Und das ist nicht die einzige offene Frage.


Ja, das ist auch das gewichtigste Gegenargument, das mir einfällt. Und es ist auch der Hauptgrund, warum ich nicht an die de-Broglie-Bohm-Theorie glaube.

amc
07.10.11, 14:00
Hallo Benjamin,

Sonst könnte man niemals einzelne Photonen durch Polarisationsfilter und dergleichen schicken, um sie nachher interferieren zu lassen.

Sofern sich hieraus beim Doppelspalt Rückschlüssse ziehen lassen auf den Weg, den das Photon genommen hat, bzw. hierdurch ein Weg definiert wird, den es genommen hat, dann gibt es keine Interferenz.

Du kannst sogar einen Strahl durch Gas schicken und den anderen durch Vakuum und du erhältst trotzdem Interferenz! Die meisten Interferenzversuche werden in Luft durchgeführt, oder glaubst du Michelson und Morley haben für ihre Versuche ein Ultrahochvakuum hergestellt?
Somit siehst du, Photonen können (einzeln!) durch Materie laufen und ergeben trotzdem ein Interferenzbild, und das obwohl sie dabei ihre Laufzeit verändern!

Wieder beim abwewandelten Doppelspalt, mit halbdurchlässigen Spiegel, und daher zwei Weg-Möglichkeiten - wenn man hier einen Weg mit einem Medium präpariert, welches einen höheren Brechingsindex besitzt, als das Medium des anderen Weges, dann wird es keine Interferenz geben, weil anhand der Laufzeitunterschiede Rückschlüsse auf den genommenen Weg zu machen sind. Oder sehe ich das falsch? Muss doch so sein?

Natürlich kann man vor den Doppelspalt einen Glasblock aufstellen, durch den das Photon läuft, und man erhält dann trotzdem ein Inteferenzbild. Und bei Michelson und Morley analogen Versuchen werden doch tatsächlich mindestens zwei Lichtimpulse gleichzeitig auf zwei verschiedene Wege geschickt, hier entsteht dann also ein Interferenzmuster trotz möglicher Laufzeitunterschiede, weil es wirklich mehrere Lichtimpulse sind die interferieren, und nicht bloß zwei mögliche Wege eines Impulses. Oder?

Grüße, AMC

Eyk van Bommel
07.10.11, 20:45
Hi JoAx,
Kannst du dein Gedankenexperiment, sein Aufbau bitte so einfach wie möglich aber exakt beschreiben? Eine Zeichnung würde auch nicht schaden.
Echt? Immer noch nicht klar? Interferometer. Weg A und Weg B. Auf dem Weg A eine Gefäß mit Gas (Chlorgas, Bromdampf..) geringer Konzentration. Auf dem Weg B Gefäß mit Raumluft (wie der Rest). Trifft ein Photon auf ein Cl2/Br2-Molekül kommt es am Schirm nicht an….nimmt es Weg B, dann?

Nochbesser wäre es man hätte anstatt dem Gefäß eine Kiste mit Schrödingers Katze. Katze tot (50%) Photon geht ohne Wechselwirkung durch die Kiste – Katze lebendig (50%) Photon wird Absorbiert.

Photon nimmt den Weg B (keiner wies es ;)) Wenn das Photon keine Interferenz aufweist, dann weist du die Katze ist tot, ohne dass jemand die Kiste aufgemacht hat:confused:

Zeichnung kommt – aber ich hatte eine Idee:) Die muss noch kurz raus (siehe „Der Kollaps der Wellenfunktion.“)

@ Benjamin

Zunächst einmal wird Licht von einem em Feld nicht beeinflusst. Es sind die Elektronen der Materie, die mit Licht in Wechselwirkung treten. Hier von einem klassischen Stoß zu sprechen, wäre zu sehr vereinfacht, wenngleich es auch solche Analogieschlüsse gibt, wo Photon-Elektron-WW wie ein relativistischer Stoßprozess von Teilchen beschrieben wird.
Aber auch so eine Beschreibung trifft die Natur nicht exakt.
Das mag sein (dass diese Beschreibung die Natur nicht exakt trifft) aber, für mich ist eine Wechselwirkung nicht automatisch mit einer Anregung verbunden? Anregung bedeutet, dass ein Impulsübertrag stattfindet. Bei einer Wechselwirkung ohne Anregung besitzen beide „Teilchen“ wieder im Betrag denselben Impuls. Bei einer Anregung ist dies nicht der Fall?
Sie breiten sich wie Wellen aus.
Ich versuch’s mal in eigenen Worten. O.K. Sie breiten sich wie Wellen aus. Die Welle ist aber eine Aufenthaltswahrscheinlichkeit. Das Photon ist ein Teilchen und dessen Aufenthaltswahrscheinlichkeit (AHW) eine Welle.
Sogar die Reflexion an einem Spiegel mit dem berühmten Gesetz, dass Einfallswinkel gleich Ausfallswinkel ist, ist eine Folge von allen möglichen Überlagerungen. Klassisch nennt man das das Huygensschen Prinzip.
Stimmt Huygenssche Prinzip, der einzige Moment in meiner Schulausbildung indem der Kauf eines Zirkels sich nicht als Sinnlos herausstellte (remember:rolleyes:)

Huygenssche Prinzip – der folgende Gedanke kam über mich wie eine Welle(das bedeutet: könnte einen eigenen Thread aufmachen)

Der Kollaps der Wellenfunktion.
Damit konnte ich nichts Anfangen. Aber wäre es falsch zusagen, die kollabierende Wellenfunktion ist ja eigentlich nur die Funktion von „n huygensschen Elementarwellen“.:rolleyes: Und da kam mir Gedanke, es ist nicht eine Welle die kollabiert. Es „kristallisiert“ sich bei einer Messung vielmehr eine heraus – alle anderen kollabieren.

Vorteil: Das kollabieren kann sich in "aller Ruhe" mit c ausbreiten ohne die Möglichkeit zur Ausbildung einer weiteren Aufenthaltswahrscheinlichkeit (AHW). Das Photon lokalisiert und von dort aus breitet sich erst der Kollaps mit c aus.
Oder eine der „n huygensschen Elementarwellen“ kollabiert (Wechselwirkung an einem Ort X) die Welle kollabiert und „reißt andere mit“. Bis zum theoretischen Treffpunkt (Schirm) dort „kristallisiert“ sich das Photon aus.

Also während das Photon (die höchst AHW) sich dem Schirm nähert, fallen immer mehr Ort als Aufenthaltswahrscheinlichkeit aus (kollabieren). Es lokalisiert immer mehr. Trifft es auf einen neuen Spiegel fängt alles von vorne an ansonsten „trifft der Kollaps“ zusammen mit dem Photon beim Schirm auf.

Meistens verstehen andere meine Gedanken erst beim 2. oder 3. mal – hoffe hier geht es schneller.

Kann ja auch hierfür eine Zeichnug machen. Oder ich bezahle Uli dafür. :D

Gruß
EVB

amc
07.10.11, 21:11
Das bezog sich auf eine ganz einfache Totalreflexion beim Einfachspalt.

Quelle ------- Spalt ---------Spiegel

Geringe Intensitaet. Bei einem Schirm statt Detektor ist Auftrittsort daher zufaellig.
Angenommen: Keine Dekohaerenz.
Ich meine der Spiegel war sogar plan.
Was passiert ?

Hi Richy,

ich werde daraus nicht wirklich schlau. Mit Einfachspalt ist also in der Tat nur ein einzelner Spalt gemeint? Wo wäre jetzt hier der Schirm im Versuchsaufbau?:confused: :)

Bis dann

richy
08.10.11, 00:21
Hi Amc

ich werde daraus nicht wirklich schlau. Mit Einfachspalt ist also in der Tat nur ein einzelner Spalt gemeint?
Ja. Schickst du ein Teilchen durch einen einfachen Spalt, so wirkt bereits hier die Aufenthaltswahrscheinlichkeit. Auch bei einem Photon. Es landet an einem zufaelligen Ort auf dem Schirm.
Quelle ----- Spalt ------- Detektorschirm

Jetzt erstetzt du den Schirm durch einen Spiegel. Und ich meine der sollte sogar plan sein. (Dann wuerde im Folgenden selbst das Reflexionsgesetz nicht gelten, sondern der Vorgang waere voellig zeitlich umkehrbar.)
Denn wenn man nun um die Quelle einen Detektor baut, kann man bestimmen wo das reflektierte Teilchen, besser dessen Psi-Welle aufrifft :

Detektorschirm
Quelle ----- Spalt ------- Spiegel
Detektorschirm

In dem Fall "kehrt" das Teilchen, dessen Psi-Welle wieder zuerueck zur Quelle.
Es trifft nicht bei den Detektoren auf.

Ich meine den Versuch koennte man schon praktisch ohne Dekohaerenz durchfuehren. Ansonsten waere das eine leere Behauptung in einer Physikvorlesung gewesen. Na und ich finde es schon erstaunlich. Der Zufall, der Ort wo das Teilchen landet ist die ganze "Zeit" dem "Teilchen" zugeordnet, vorbestimmt. Zeit und "Teilchen" in Anfuehrungszeichen, weil in der isolierten Umgebung weder ein geordneter Zeitablauf noch ein reales Teilchen existiert. Das kann sich in seinem isolierten Bezugssystem daher nicht von der Quelle zum Schirm oder Spiegel in klassischer Weise fortbewegen. Das d/dt in der SGL ist ein d/dt fuer die geordnet ablaufende Zeit eines Beobachters.
Man muesste somit nicht nur den Wellencharakter sondern auch dessen vorbestimmter zufaelliger Auftrittsort dem "virtuellen Teilchen" zuordnen.

Gruesse
PS: Ich schau mal ob ich das Video finde, falls es noch nicht klar ist.

JoAx
08.10.11, 11:35
Hallo Benjamin!


Demnach durchlaufen Photonen definitiv Glas oder Gas, ohne mit ihm in WW zu treten. Sonst könnte man niemals einzelne Photonen durch Polarisationsfilter und dergleichen schicken, um sie nachher interferieren zu lassen. Photonen können durch Materie präpariert werden, ohne dass sie dabei Energie abgeben!


Was?!?
Das scheint mir nicht haltbar zu sein, was du da sagst.
Bsp.: Diamant/Graphit. Beide bestehen (idealisiert) nur aus Kohlenstoff. Das eine ist durchsichtig, das andere nicht.
- Warum?
- Durchläuft das Licht (teilweise) den Diamanten, ohne mit ihm wechsel zu wirken?
- Und das trotz höherer Dichte? Das würde ich bezweifeln.
- Ist Materie Energielos? (Bindungen in/zwischen Molekülen.)
- Kann ein Photon in der Materie nicht so gestreut werden, dass es "einfach weitergereicht" wird? (Die Ursprüngliche Richtung und Energie beibehalten wird.)


Ich zeig dir an einem einfachen Beispiel, dass deine Überlegungen falsch sind: Du kannst Photonen sowohl durch Luft als auch durch Glas schicken und erhältst trotzdem einwandfreie Interferenz. Du kannst sogar einen Strahl durch Gas schicken und den anderen durch Vakuum und du erhältst trotzdem Interferenz!


Ok. - Warum "funktioniert" das?


oder glaubst du Michelson und Morley haben für ihre Versuche ein Ultrahochvakuum hergestellt?


Bei dem Interferometer-Typ, so wie die Authoren ihn genutzt haben, ist sowieso die klassische ED anwendbar.


Weil nur die Möglichkeit einer WW - also die Möglichkeit von Energieübertragung auf die Materie - die Bewegung des Photons beeinflussen.


Nicht einverstanden. :)


Gruß, Johann

JoAx
08.10.11, 11:52
Hallo Eyk!


Echt? Immer noch nicht klar? Interferometer. Weg A und Weg B. Auf dem Weg A eine Gefäß mit Gas (Chlorgas, Bromdampf..) geringer Konzentration. Auf dem Weg B Gefäß mit Raumluft (wie der Rest).


Ok. So eine Art Fizeau-Interferometer.

http://upload.wikimedia.org/wikipedia/commons/thumb/c/c0/Fizeau-Mascart1.png/800px-Fizeau-Mascart1.png


Trifft ein Photon auf ein Cl2/Br2-Molekül kommt es am Schirm nicht an….nimmt es Weg B, dann?


Wenn das Photon auf dem Weg A in (nahe zu) 100% der Fälle absorbiert wird, dann dürfte keine IF zu sehen sein. (Die armen Katzen würde ich in Ruhe lassen. :))


Gruß, Johann

Eyk van Bommel
08.10.11, 12:54
Wenn das Photon auf dem Weg A in (nahe zu) 100% der Fälle absorbiert wird, dann dürfte keine IF zu sehen sein.
Keine Frage. Denn jeder würde wissen, dass das Photon das hinten ankommt den Weg B genommen haben muss. Über A kommt ja nichts an.

Aber was ist bei 50% Absorption? Wir als Beobachter werden nicht wissen, war es eines von denen 100% über den Weg B oder eines von den 50% über den Weg A?

Keine „welcher Weg“ Information. Aber 50% der Photonen über B „spüren“ sie wären gemessen worden?

Also insgesamt nur 50% Interferenz bei den anderen 50 keine ? Oder 75%?

Gruß
EVB

Weil nur die Möglichkeit einer WW - also die Möglichkeit von Energieübertragung auf die Materie - die Bewegung des Photons beeinflussen.
Nicht einverstanden.
Bei 50% Interferenz wäre diese Aussage aber richtig? Oder?

amc
08.10.11, 13:50
Aber was ist bei 50% Absorption? Wir als Beobachter werden nicht wissen, war es eines von denen 100% über den Weg B oder eines von den 50% über den Weg A?

Keine „welcher Weg“ Information. Aber 50% der Photonen über B „spüren“ sie wären gemessen worden?

Also insgesamt nur 50% Interferenz bei den anderen 50 keine ? Oder 75%?

Hallo Eyk,

ich würde auf 75% tippen. Die Wahrscheinlichkeit der Wege beträgt jeweils 50%. Dazu werden bei Weg Links (Weg A) 50% absorbiert - also 25% absolut, bleiben 75% die durchkommen. Für diese gilt: Weg Rechts (Weg B) die dopppelte Wahrscheinlichkeit als bei Weg Links. 50% zu 25%.

Ich denke also, für die 75% die durchkommen, ist natürlich der Weg nicht definiert (es gibt also keine, die Weg A oder B genommen haben, außer diejenigen, die absorbiert wurden), deswegen gibt es Interferenz. Aber das Muster wird rechtsseitig ausgeprägter sein, weil dieser Weg doppelt wahrscheinlich ist.

Zustimmung?

Grüße, AMC

Eyk van Bommel
08.10.11, 14:31
Hallo amc,
Nur zum Aufbau:

Hier gibt es keinen Doppelspalt. Es ist ein „normales" Mach-Zehnder-Interferometer (z.B. http://www.didaktik.physik.uni-muenchen.de/materialien/inhalt_materialien/milq/interferometer.pdf).


Stelle dir den grauen Kasten auf Wiki (http://de.wikipedia.org/wiki/Mach-Zehnder-Interferometer)als gasgefüllten Zylinder vor.

Gruß
EVB

PS: Bei 100% Absorptionswahrscheinlichkeit auf dem Weg A kommen 100% der Photonen über Weg B an. Alle Photonen über Weg B werden keine Interferenz aufweisen!

Eyk van Bommel
08.10.11, 17:08
Die armen Katzen würde ich in Ruhe lassen

Habe ich gerade gefunden JoAx

Wechselwirkungsfreie Quantenmessung (http://de.wikipedia.org/wiki/Wechselwirkungsfreie_Quantenmessung)

Die Katze lebt!!!:eek: Ähm - oder auch nicht:o

;)

Eyk van Bommel
08.10.11, 17:39
Schaut mal hier (http://homepage.univie.ac.at/franz.embacher/dasDing/video.html).

Nach dem 1/3 wird das ganz gut erklärt. Die Photon spüren, dass was im Weg gewesen wäre:cool:

Benjamin
08.10.11, 19:13
Was?!?
Das scheint mir nicht haltbar zu sein, was du da sagst.
Bsp.: Diamant/Graphit. Beide bestehen (idealisiert) nur aus Kohlenstoff. Das eine ist durchsichtig, das andere nicht.
- Warum?
- Durchläuft das Licht (teilweise) den Diamanten, ohne mit ihm wechsel zu wirken?
- Und das trotz höherer Dichte? Das würde ich bezweifeln.
- Ist Materie Energielos? (Bindungen in/zwischen Molekülen.)
- Kann ein Photon in der Materie nicht so gestreut werden, dass es "einfach weitergereicht" wird? (Die Ursprüngliche Richtung und Energie beibehalten wird.)

Die Dichte eines Materials hat nur bedingt mit seinen optischen Eigenschaften zu tun. In der Tat ist es so, dass diese Eigenschaften sich aus den Möglichkeiten ergeben, die Licht hat, dort zu reagieren. Kann Licht keine Energiezustände anregen, läuft es durch das Material durch.

Ok. - Warum "funktioniert" das?

Weil Photonen durch Materie laufen können, ohne mit ihr in Wechselwirkung zu treten.

Nicht einverstanden.

Auch wenn du damit nicht einverstanden bist, ist es dennoch so.

amc
08.10.11, 19:16
Wechselwirkungsfreie Quantenmessung (http://de.wikipedia.org/wiki/Wechselwirkungsfreie_Quantenmessung)

Sehr interessant. Allerdings im Grunde auch nichts Weltbewegendes (mal abgesehen davon, dass die QM verrückt ist, aber das wissen wir ja schon :) ). Finde ich zumindest ...

Wechselwirkungsfreie Quantenmessung halte ich für etwas gewagt und auch irreführend, weil es ja im Grunde, einem normalen Doppelspalt entspricht, bei dem ein Spalt verschlossen ist. Hier kann man dann auch, ohne den Spalt direkt sehen bzw. messen zu müssen, einfach anhand des Ergebnisses am Schirm erkennen, ob ein Objekt im Weg ist (ein Spalt verschlossen ist). Der Prozess als solches, im Ganzen, ist als Messprozess anzusehen. Anhand des Ergebnisses erhält man ggf. Informationen und Rückschlüsse über den Aufbau des gesamten Versuchsapparates.

Die Photon spüren, dass was im Weg gewesen wäre:cool:

(Hab mir das Video noch nicht angeschaut)

Genau. Es zeigt sich hier nämlich wunderbar der nichtlokale Charakter der QM ( zumindest so wie ich das Ganze verstehe). Wie Benjamin schon sagte, hat alleine die Möglichkeit einer WW bereits Einfluss auf das Verhalten des Quantenobjektes. Und zwar ohne Kommunikationsaustausch, instantan und nichtlokal. Ebenso zu sehen beim normalen Doppelspalt, wenn ein Spalt verschlossen ist. Hier "weiß" das Photon auch, ein Spalt ist verschlossen, also rauscht es mit Teilchencharakter durch den anderen Spalt. Für diese Art Versuche eignen sich natürlich Aufbauten a la Mach-Zehnder-Interferometer viel besser als normale Doppelspalte, weil man mehr und genauere Möglichkeiten hat zu präparieren, zu justieren, und auszuwerten.

Grüße, AMC

JoAx
09.10.11, 00:50
Hallo Benjamin!


Die Dichte eines Materials hat nur bedingt mit seinen optischen Eigenschaften zu tun.


Das ist mir schon klar.


In der Tat ist es so, dass diese Eigenschaften sich aus den Möglichkeiten ergeben, die Licht hat, dort zu reagieren.


Das ist eine zu pauschale Antwort.


Kann Licht keine Energiezustände anregen, läuft es durch das Material durch.


"Läuft durch" klingt so, als ob da nix wäre. Ist aber. Sonst würde die Geschwindigkeit sich auch nicht ändern. Warum/Wozu auch? Man kann zwar nicht vorhersagen, wie ein ausgewähltes Photon gestreut wird, aber z.B., dass 90% in die selbe Richtung wieder abgestrahlt werden, in die sie zuvor "geflogen" sind. Oder, dass 50% unter 90°, oder ... . Das hängt dann u.a. auch vom Winkel ab, unter dem der Strahl auftrifft. Es spielen dabei also nicht nur (bzw. manchmal gar nicht) die Energiezustände der Atome selbst eine Rolle.
Oder sehe ich da was falsch?


Weil Photonen durch Materie laufen können, ohne mit ihr in Wechselwirkung zu treten.


In welchem Experiment wurde das nachgewiesen?
In welchem Experiment hat man beobachtet, wie ein Photon "läuft"?


Auch wenn du damit nicht einverstanden bist, ist es dennoch so.

Wenn du das sagst.

Alles IMHO (zu unserer Ausgangsfrage) ->
Es geht darum, eine Aussage zu formulieren, auf die man sich, ähnlich dem Postulat über die Konstanz der LG im Vakuum, 100% verlassen kann. Die "interpretationsneutral" wäre, und definitiv nichts falsches, respektive spekulatives enthalten würde. Deine Ausführungen sind es im Moment imho nicht.


Gruß, Johann

JoAx
09.10.11, 01:48
Hallo Eyk!


Also insgesamt nur 50% Interferenz bei den anderen 50 keine ? Oder 75%?


Ich weiß nicht, ob die Überlegung und "Rechnung" so richtig sein wird:
100% der Photone gehen durch beide Spalte.
25% werden gestoppt.
50% gehen definitiv durch, ohne Info. über den genommenen Weg zu hinterlassen -> Interferenz.
Bleiben also 25%, die nicht interferieren dürften. Und sie alle kommen über B.

???


Gruß, Johann

richy
09.10.11, 02:09
Hi Joax
Um die Wellenfunktion des Photons zu verändern, reicht allein die Möglichkeit einer Wechselwirkung, sprich Messung aus.

Die "interpretationsneutral" wäre,
Es ist nunmal die Schulmeinung. Und da hier vielleicht auch mal Schueler mitlesen finde ich Benjamins Aussage in Ordnung.
Beim Dekohaerenzprogamm der VWI wuerde man dies vielleicht anders betrachten.

BTW:
Ein internationales Forscherteam nutzte im Juni 2007 zum ersten Mal die von Einstein postulierte „spukhafte Fernwirkung" über eine Distanz von 144 Kilometern - zehn mal weiter als je zuvor gemessen. Sie erzeugten auf der Kanarischen Insel Las Palmas zwei verschränkte Photonen und schickten dann eines der Photonen über ein Teleskop auf seine 144 Kilometer lange Reise durch die Luft zum Empfängerteleskop auf der Nachbarinsel Teneriffa. Wenn beide Photonen durch Messungen aus ihrer Überlagerung herausgerissen wurden, entschieden sie sich mit hoher Wahrscheinlichkeit in La Palma wie auf Teneriffa für die gleiche Polarisation – ihr Zustand wurde damit erfolgreich übertragen.

http://www.scinexx.de/dossier-detail-374-2.html
Scheint bei Photonen gar nicht so kritisch zu sein, so wie man sich das immer vorstellt.

Gruesse

Benjamin
09.10.11, 02:24
Alles IMHO (zu unserer Ausgangsfrage) ->
Es geht darum, eine Aussage zu formulieren, auf die man sich, ähnlich dem Postulat über die Konstanz der LG im Vakuum, 100% verlassen kann. Die "interpretationsneutral" wäre, und definitiv nichts falsches, respektive spekulatives enthalten würde. Deine Ausführungen sind es im Moment imho nicht.


Meine Aussagen sind fundamentale QED und damit Teil einer der genauesten geprüften Theorien, die der Mensch je über die Natur hatte.

JoAx
09.10.11, 02:58
Hi Benjamin und richy!

Meine Aussagen sind fundamentale QED und damit Teil einer der genauesten geprüften Theorien, die der Mensch je über die Natur hatte.

Ich kann mich sehr gut irren, oder auch etwas missverstehen.

Ich denke, es wäre zunächst einmal wichtig "Wechselwirkung" zu definieren. Ich verstehe unter WW einen physikalischen Vorgang, wo Energieaustausch bzw. -umwandlung stattfindet. Das ist auch so ziemlich die gängigste Definition in der Physik.

Demnach durchlaufen Photonen definitiv Glas oder Gas, ohne mit ihm in WW zu treten. Sonst könnte man niemals einzelne Photonen durch Polarisationsfilter und dergleichen schicken, um sie nachher interferieren zu lassen. Photonen können durch Materie präpariert werden, ohne dass sie dabei Energie abgeben!


Daraus drängt es mich abzuleiten, dass du sagen möchtest, dass eine Präparation wechselwirkungsfrei abläuft.
Ist das richtig?


Präparation und Messung in der Quantenmechanik

Die theoretische Deutung eines Experimentes erfordert gemäß Heisenberg drei Schritte[1]:

1. Es wird die experimentelle Ausgangssituation vorbereitet (Präparation) und durch eine Wellenfunktion ψ beschrieben.
2. Es findet eine zeitliche Entwicklung des physikalischen Systems statt. Im sog. Schrödingerbild[2] verändert sich dabei nur die Wellenfunktion. Diese Veränderung wird durch die Schrödingergleichung beschrieben.
3. Es wird die physikalische Messung am System vorgenommen (Registrierung). Aufgrund der Präparation kann die Wahrscheinlichkeit für das Ergebnis dieser Registrierung aus der Wellenfunktion vorhergesagt werden.
Besonders Lamb verweist in diesem Zusammenhang auf eine eindeutige Klärung, worin der erste Schritt des Prozesses genau bestehen soll, also nach Bohr die Fixierung der äußeren Bedingungen, durch welche die Ausgangssituation des betrachteten Systems definiert wird (z. B. ein Teilchen befindet sich mit Wahrscheinlichkeit 1 im Intervall der Länge Δx > 0).

Die Unterscheidung zwischen Präparation und Messung wird aus dem Charakter der jeweiligen Ergebnisse der beiden Schritte ersichtlich. Das Ergebnis einer einzelnen Messung ist ein Wert der Observablen (oder eine Eigenschaft). Bei Wiederholungen der Messung erhält man nicht dasselbe Ergebnis wie zuvor, sondern bestenfalls eine Werte-Statistik für die fragliche Observable. Hingegen liefert die Präparation ein vollständiges Wissen davon, mit welcher Wahrscheinlichkeit die betrachtete Observable einen Wert (oder eine Eigenschaft) haben kann, und das für jede Observable[3].

Wenn in der Literatur von zwei unmittelbar aufeinanderfolgenden Messungen die Rede ist, so handelt es sich dabei oft um eine Präparation mit anschließender Registrierung des Messobjektes. Oft wird dabei bereits der Kenntnissgewinn durch die anfängliche Präparation als Messergebnis verwendet.


Hier sehe ich dagegen nichts, was darauf hinweisen würde, dass eine Präparation ww-frei abzulaufen hat. Und wenn ich das mit dem vergleiche, was auch richy von dir zitiert hat:

Um die Wellenfunktion des Photons zu verändern, reicht allein die Möglichkeit einer Wechselwirkung, sprich Messung aus.

dann drängt es mich wieder, dir (und richy) zu widersprechen :o
Der Unterschied zwischen Präparation und Messung liegt offenbar gar nicht in vorhandener oder nicht vorhandener Wechselwirkung.

Ehrlich - Was verstehe ich falsch?


Gruß, Johann

Eyk van Bommel
09.10.11, 08:54
Hallo Eyk!



Ich weiß nicht, ob die Überlegung und "Rechnung" so richtig sein wird:
100% der Photone gehen durch beide Spalte.
25% werden gestoppt.
50% gehen definitiv durch, ohne Info. über den genommenen Weg zu hinterlassen -> Interferenz.
Bleiben also 25%, die nicht interferieren dürften. Und sie alle kommen über B.

???


Gruß, Johann

ja-so sehe ich das auch!

amc
09.10.11, 09:19
ja-so sehe ich das auch!

Moin,

es tut mir leid ich muss widersprechen. Ausgang war: Auf Weg A befindet sich eine Kammer, die zu 50% die Photonen schluckt, richtig?

50% gehen links durch, 50% rechts, rein statistisch, würde man messen! Also 25% absolut werden auf Weg A gestoppt. Soweit sind wir uns einig, oder?

Bleiben 75% übrig. Wodurch sollte denn für diese 75% definiert sein welchen Weg sie genommen haben? Es ist nicht definiert, definiert ist lediglich, die Wahrscheinlichkeit von Weg B liegt doppelt so hoch wie Weg A. Es müssten also alle 75% inteferieren, das ungleiche Wahrscheinlichkeitsverhältnis jedoch wird sich auf das Interferenzmuster (bzw. Interferenzverhalten am zweiten halbdurchlässigen Spiegel) auswirken. Das Interferenzergebnis wird also ein anderes sein müssen, als wenn die Wahrscheinlichkeiten der Wege genau gleich sind.

Also, anders formuliert: Warum sollten sich 25% der Photonen für Weg B entscheiden, wenn sie dies aber gar nicht müssen?

Ich kann natürlich falsch liegen. Für mich macht es jedoch zur Zeit nur so Sinn.

Grüße, AMC

Eyk van Bommel
09.10.11, 09:38
Wodurch sollte denn für diese 75% definiert sein welchen Weg sie genommen haben?
Da 50% der Photonen die den Weg B gehen „spüren“, dass sie auf dem Weg A gemessen worden wären, werden diese sich als Teilchen verhalten.
Also 25% absolut werden auf Weg A gestoppt.
Richtig - Und weitere 25% auf dem Weg B kommen daher als Teilchen an! Die, die bei A gemessen worden wären, wären sie den Weg A gegangen, was sie nicht sind, da sie auf dem Schirm als Teilchen aufschlagen!

Gruß
EVB

JoAx
09.10.11, 10:24
Hallo amc!


Es ist nicht definiert, definiert ist lediglich, die Wahrscheinlichkeit von Weg B liegt doppelt so hoch wie Weg A.


Richtig. D.h. -

1. WegA + WegB = 75%
2. WegB verhält sich zu WegA wie 2 zu 1,
50%:25% = 2:1

Du sagst nun, alle 75% interferieren. Machen wir eine Prüfung. Würden alle Photone, die WegA genommen haben absorbiert werden, dann entspräche das dem Fall, dass eines der Löcher immer verschlossen ist. Wir hätten also nur ein Loch. Nach deinen Überlegungen müsste man erwarten, dass das "2Loch-Interferenzbild" dennoch da ist, nur mit in Richtung WegB verschobenen Zentrum. Kann das stimmen?

@Eyk
Als Teilchen kommen sie alle an. :)


Gruß, Johann

Benjamin
09.10.11, 10:30
dann drängt es mich wieder, dir (und richy) zu widersprechen :o
Der Unterschied zwischen Präparation und Messung liegt offenbar gar nicht in vorhandener oder nicht vorhandener Wechselwirkung.

Ehrlich - Was verstehe ich falsch?

Wie gesagt, hängt es davon ab, wie du WW definierst. Es ist freilich falsch, zu sagen, Licht läuft durch Glas ohne mit ihm in WW zu treten, alleine deshalb schon, weil Licht auch Energie aufs Glas überträgt. Es lässt sich nun darüber streiten, ob es angebracht ist, zu sagen, ein Photon kann durch Glas laufen, ohne mit ihm wechselzuwirken. Fakt ist, es kann zumindest durch Glas laufen, ohne Energie abzugeben. Ich will mich aber nicht auf Wortklauberei einlassen, bleiben wir lieber bei der Physik. Und dazu fällt mir folgende Erklärung ein:

Aus der Dirac-Gleichung folgt exakt der Wert 2 für den Landé-Faktor. Experimentell beobachtet man eine gerine Abweichung von diesem Wert, so hat er etwa den Wert 2,0023... Warum das so ist, wird erst in der QED zufriedenstellend aufgelöst. Und zwar kommt die Abweichung daher, weil die Diracsche Betrachtung die WW des Elektrons mit seinem eigenen Feld nicht einbezieht. Um nun diese WW zu berechnen, sieht man sich alle Möglichkeiten an, die das Elektron hat mit dem em Feld in WW zu treten. Das kann zB darin bestehen, dass das Elektron ein Photon aufnimmt, oder dass es eines abgibt, oder dass es eines abgibt und gleich wieder aufnimmt, oder dass es eines abgibt, welches dann spontan ein Elektron-Positron-Paar bildet, welches wieder zu einem Photon annihiliert, oder dass das Elektron ein Photon emittiert, welches hitnereinander zwei Elektron-Positron-Paare bildet, usw.
Um also das Verhalten des Elektrons zu beschreiben, sehen wir uns alle Möglichkeiten an, die geschehen können, und Summieren ihre Wahrscheinlichkeitsamplituden gemäß den Feynmenschen Regeln und mithilfe von Pfadintegralen.

Natürlich gibt es natürlich unendlich viele solcher Möglichkeiten, glücklicherwiese können wir jedoch gut zwischen wahrscheinlichen und weniger wahrscheinlichen Vorgängen unterscheiden. Damit, dass wir uns einfach die wahrscheinlichsten herausgreifen, gelingt eine sehr gute Näherung mit - sagen wir - 1000 oder 10000 betrachteten Möglichkeiten, was selbstverständlich nur mehr mit dem Computer ausgewertet werden kann.

Letztlich ist also das reale Verhalten des Elektrons daran zu messen, welche Möglichkeiten es theoretisch hat.

Eyk van Bommel
09.10.11, 10:59
Hallo Benjamin,

gibt es bei dir keinen Unterschied zwischen Wechselwirkung und Anregung? Für mich ist die Anregung nur eine Form der verschiedenen Arten von verschiednen Arten der möglichen Wechselwirkungen, die ein Quantenobjekt/Teilchen eingehen kann.

Und nicht jede Form der Wechselwirkung ermöglicht auch ein Informationsgewinn - oder?

Man kann in einem Interferometer auch zwei Glaszylinder einbauen. Einen auf dem Weg A und einen auf dem Weg B – hier kommt zu ~100% zur Interferenz-oder? Da das Licht aber diesen Aufbau langsamer durchquert wie im Vakuum, kann man eine Wechselwirkung mit dem Glas annehmen.

Offenbar reicht doch alleine die Möglichkeit zur Wechselwirkung nicht aus? Es muss eine Anregung sein? Oder eine Wechselwirkung die zu einem Informationsgewinn führt bzw. führen würde? Bei einem Glaszylinder in nur einem der beiden Wege, wäre Lichtlaufzeit – hier ist es nicht die Wechselwirkung, sondern die Aufenthaltswahrscheinlichkeitswelle trifft auf dem einen Weg später auf, wie auf dem anderen Weg. Das identisch mit der Verlängerung der Strecke bei einem der beiden Wege.

Gruß
EVB

PS: Kannst du der tatsächlich „Führungswelle-Theorie“ etwas abgewinnen?

amc
09.10.11, 11:16
Würden alle Photone, die WegA genommen haben absorbiert werden, dann entspräche das dem Fall, dass eines der Löcher immer verschlossen ist. Wir hätten also nur ein Loch. Nach deinen Überlegungen müsste man erwarten, dass das "2Loch-Interferenzbild" dennoch da ist, nur mit in Richtung WegB verschobenen Zentrum. Kann das stimmen?

Moin Johann,

nein das kann so nicht stimmen, aber das sage ich ja so auch nicht. Wenn 100% auf Weg A verschluckt werden, dann bleibt nur Weg B - ergo der Weg ist definiert -> Teilchencharakter. Wenn es allerdings, auch nur eine noch so geringe Wahscheinlichkeit dafür gibt, dass es Weg A oder B genommen haben könnte, dann ist für das einzelne Photon der Weg nicht mehr exakt definiert. Für jedes einzelne der 75% muss dann, nach meiner Überzeugung, gelten, 33,33% Wahrscheinlichkeit für Weg A und 66,66% Wahrscheinlichkeit für Weg B. Diese Aussage muss für jedes einzelne dieser Photonen gelten, daher ist jedes als gleichwertig anzusehen, ergo interferieren alle.

Allerdings müsste dies, ähnlich wie ich dich verstehe, schon zu dem Schluss führen, dass niemals eine tatsächlich 100% gesicherte Aussage darüber zu machen ist, ob sich ein Photon als Teilchen oder als Welle verhalten hat. Auch wenn ein Spalt verschlossen ist, dann gibt es doch immer eine möglicherweise noch so minimale Wahrscheinlchkeit dafür, dass es trotzdem WW-frei durch den verschlossenen Spalt hindurchrauscht. Es wird sich also in der Tat niemals zu 100%, sondern oft z.B. nur zu 99,9999% sagen lassen, ob das einzelne Photon sich jetzt als Welle oder Teilchen verhalten halt.

Im Prinzip, wenn man dies auf deine obige Aussage übertragt, bedeutet es tatsächlich, dass es im Grunde immer Interferferenzerscheinungen sind, die wir indirekt beobachten, diese aber, wenn die Wege der einzelnen Quantenobjekte als nahezu definiert anzusehen sind, für uns, von einem aus Sicht der klassischen Mechanik zu erwartenden Teilchenmuster, nicht zu unterscheiden sind.

Grüße, AMC

JoAx
09.10.11, 11:57
Hallo amc!


Wenn es allerdings, auch nur eine noch so geringe Wahscheinlichkeit dafür gibt, dass es Weg A oder B genommen haben könnte, dann ist für das einzelne Photon der Weg nicht mehr exakt definiert. Für jedes einzelne der 75% muss dann, nach meiner Überzeugung, gelten, 33,33% Wahrscheinlichkeit für Weg A und 66,66% Wahrscheinlichkeit für Weg B. Diese Aussage muss für jedes einzelne dieser Photonen gelten, daher ist jedes als gleichwertig anzusehen, ergo interferieren alle.


Ok. Ein anderes Beispiel. Ich zeige dir ein Bild, das zu einem DS-Experiment gehört. Und sage dir, dass bei all den einzeln durchgeschickten Photonen (sehr vielen), das 2. Spalt nur ein Mal (zufällig) tatsächlich geöffnet war. Denkst du, dass du es an diesem Bild wirst ablesen können? Ein Mal war es ja offen. :)
Wird sich dieses Bild von einem unterscheiden, in dem es die ganze Zeit nur einen Spalt gab?

http://upload.wikimedia.org/wikipedia/de/thumb/8/8c/Doppelspalt_muster.png/640px-Doppelspalt_muster.png

Die QM ist irre. :D


Gruß, Johann

Eyk van Bommel
09.10.11, 12:15
Hallo amc,

schaue dir das Video doch mal an. Wie gesagt, nach ca. 1/3 (das Vordere habe ich nicht angeschaut ;)) wird genau dieses Beispiel behandelt.

Das Photon nimmt den Weg B! Warum? Keiner (kein Beobachter wusste, ob die Bombe explodieren kann oder nicht – Hinweis: Schrödingers Katze. ) Hätte das Photon eine Interferenz gezeigt, hätte auch keiner gewusst, dass es den Weg B genommen hat.

Obwohl keiner die Information „welcher Weg“ erhalten hätte!!! Zeigt das Photon keine Interferenz!

Gruß
EVB

PS:
Zitat JoAx: Die QM ist irre.
Ja schon :) Aber wäre sie nicht noch verrückter, wenn das System wissen würde, dass man es (eigentlich) nicht herausfinden kann und daher eine Interferenz aufweisen?

Obwohl hier keine „welcher Weg“ Information vorliegt – zeigt das Photon keine Interferenz!

Eine "welcher Weg" Information liegt vor = keine Interferenz

Aber

keine Interferenz = eine "welcher Weg" Information liegt vor: Stimmt nicht!!

amc
09.10.11, 12:54
Denkst du, dass du es an diesem Bild wirst ablesen können? Ein Mal war es ja offen. :)
Wird sich dieses Bild von einem unterscheiden, in dem es die ganze Zeit nur einen Spalt gab?

Ist es nicht genau das, was ich sage? Es muss immer eine hinreichende Sigifikanz vorherschen, damit wir zwischen Wellen- und Teilchenverhalten unterscheiden können. Das minimal vorhandene Inteferenzmuster, muss in dem Ergebnis enhalten sein, wir können es aber nicht erkennen, weil unsere Experimente immer nur näherungsweise Ergebnisse liefern. Dies reicht ja auch im Grunde aus. Wofür müssen wir wissen, ob z.B. eines von einer Trilliarde Photonen möglicherweise Wellencharakter gezeigt hat? Das können wir gar nicht wissen. Und das wäre, schätze ich, meistens auch nur für theoretische Überlegungen hilfreich.

Grüße, AMC

JoAx
09.10.11, 13:09
Hallo Eyk!


Obwohl hier keine „welcher Weg“ Information vorliegt – zeigt das Photon keine Interferenz!


Eigentlich schon. Wir wissen ja, dass die Photone, die durchkommen aber keine Interferenz zeigen, nur durch WegB zum Schirm gelangen konnten.


Gruß, Johann

amc
09.10.11, 13:11
Das Photon nimmt den Weg B! Warum? Keiner (kein Beobachter wusste, ob die Bombe explodieren kann oder nicht – Hinweis: Schrödingers Katze. ) Hätte das Photon eine Interferenz gezeigt, hätte auch keiner gewusst, dass es den Weg B genommen hat.

Obwohl keiner die Information „welcher Weg“ erhalten hätte!!! Zeigt das Photon keine Interferenz!

(Hab mir das Video immer noch nicht angeschaut :) )

Aber dort geht es doch um einen analogen Fall, wie er hier, in diesem von dir angeführten Wiki-Eintrag beschrieben wird?

http://de.wikipedia.org/wiki/Wechselwirkungsfreie_Quantenmessung

Also, Weg A ist entweder mit einem Objekt versehen, welches zu 100% die Photonen verschluckt, oder es ist nicht da. Dieser Fall ist eindeutig. Hier werden auf Weg A 100% verschluckt, also 50% absolut. Für die restlichen 50% bleibt jetzt nur noch mit Teilchencharakter durch Weg B zu rauschen, und hiermit, die Anwesenheit des Objektes auf Weg A indirekt nachzuweisen.

Nur wir reden ja jetzt über den Fall, wenn das Objekt auf Weg A nur 50% der Photonen schluckt, also 25% absolut. Nach eurer Ansicht werden durch die Schluckung der 25% auf Weg A auch 25% der Photonen gezwungen Weg B als Teilchen zu passsieren. Bleiben dann also noch 50% übrig, die interferieren. Ich bezweifle dies immer noch, ich sage 75%, wegen der Wahrscheinlichkeit von 2:1 verändert sich jedoch das Interferenzergebnis :)

Man könnte es ganz leicht experimentell überprüfen: Nehmen wir den Aufbau aus dem genannten Wiki-Eintrag. Dieses Mach-Zehnder-Interferometer ist so eingestellt, dass bei Interferenz nur der rechte Detektor anschlägt. So, nun platzieren wir jeweils auf Weg A und auf Weg B ein Objekt, welches jeweils 50% der Photonen schluckt. Nach eurer Meinung dürfte es jetzt zu keiner Interferenz mehr kommen, da 25% auf Weg A geschluckt werden, und 25% gezwungen werden als Teilchen Weg B zu nehmen, ebenfalls werden 25% auf Weg B geschluckt, sowie 25% gezwungen als Teilchen Weg A zu nehmen. Also im Ergebnis, nach eurer Meinung, werden 50% absolut geschluckt, und jeweils 25% werden gezwungen als Teilchen Weg A oder B zu nehmen -> keine Interferenz -> beide Detektoren schlagen an. Ich sage: 50% werden geschluckt und 50% interferieren -> nur rechter Detektor schlägt an.

Wer macht den Versuch? ;)

Grüße, AMC

Eyk van Bommel
09.10.11, 15:34
Aber dort geht es doch um einen analogen Fall, wie er hier, in diesem von dir angeführten Wiki-Eintrag beschrieben wird?
Nein;)
Hab mir das Video immer noch nicht angeschaut
Da kommst'e nicht drum rum:)
Es geht um 50%/50% und zwar im Sinne von Schrödingers Katze!

Schaue dir das mit der "Bombe" mal an.

JoAx
09.10.11, 17:16
Hi AMC!


Ich bezweifle dies immer noch, ich sage 75%,


75% interferieren heißt in diesem Experiment, dass 100% von denen, die durchkommen/detektiert werden, im rechten Detektor landen. => Keins landet im oberen.
Tatsächlich wird es imho so sein, dass 2/3 im rechten landen, und 1/3 im oberen. => 1/3 (=25% der Intensität der Kanone) interferiert gar nicht.

Oder?


So, nun platzieren wir jeweils auf Weg A und auf Weg B ein Objekt, welches jeweils 50% der Photonen schluckt. Nach eurer Meinung dürfte es jetzt zu keiner Interferenz mehr kommen, da 25% auf Weg A geschluckt werden, und 25% gezwungen werden als Teilchen Weg B zu nehmen, ebenfalls werden 25% auf Weg B geschluckt, sowie 25% gezwungen als Teilchen Weg A zu nehmen. Also im Ergebnis, nach eurer Meinung, werden 50% absolut geschluckt, und jeweils 25% werden gezwungen als Teilchen Weg A oder B zu nehmen -> keine Interferenz -> beide Detektoren schlagen an. Ich sage: 50% werden geschluckt und 50% interferieren -> nur rechter Detektor schlägt an.


Ich hätte so angesetzt:
1. Nichts kommt an -> 50%
2. WegA ohne Interferenz -> 12,5%
3. WegB ohne Interferenz -> 12,5%
3. WegA oder WegB mit Interferenz -> 25%

Wir hätten am Schirm also eine Mischung aus drei Varianten.
25% - Interferenz
12,5% - keine Interferenz und WegA
12,5% - keine Interferenz und WegB

Aber der Interferometer könnte das vermutlich so nicht auflösen.
???

Gruß, Johann

amc
09.10.11, 17:28
Nein;)

Oh doch Eyk!

Da kommst'e nicht drum rum:)

Genau das hab ich jetzt gemacht. Es geht dabei genau wie hier: http://de.wikipedia.org/wiki/Wechselwirkungsfreie_Quantenmessung#Elitzur-Vaidman-Schema - um das Elitzur-Vaidman-Schema. Also ob man auch, ohne direkt zu messen, herausbekommen kann, ob ein Hindernis den einen Weg blockiert. Und zwar eines, das, wenn es denn vorhanden ist, zu 100% die Photonen blockiert, die diesen Weg nehmen!

Und da, wenn das Objekt vorhanden ist, es nur zwei Möglichkeiten für das einzelne Photon gibt, nämlich zu 50% auf das Objekt zu treffen und absorbiert zu werden (oder abgelenkt), oder zu 50% mit Teilchencharakter Weg B zu nehmen, kann man nun, wenn der obere Detektor anschlägt, sicher sein, dass Objekt ist vorhanden, obwohl man es nicht direkt gemessen hat.

Und warum geht das? Wie ich schon sagte, und wie im Video erklärt wird: Nichtlokalität!!!

Grüße, AMC

P.S.: Wie sich der Versuch verhält, wenn ein Objekt platziert wird, welches zu 50% die Photonen schluckt, bleibt hier also weiterhin offen!

amc
09.10.11, 17:30
75% interferieren heißt in diesem Experiment, dass 100% von denen, die durchkommen/detektiert werden, im rechten Detektor landen. => Keins landet im oberen.

Nein das heißt es nicht, nur dann, wenn die Wahrscheinlichkeiten der Wege identisch sind. Wenn nicht, dann wird es zu Phasenverschiebungen kommen!

Grüße, AMC

richy
09.10.11, 17:33
Hi
Mir ist das zu hoch. Gehts auch einfacher ?
Stellt das Bild des Einzelspaltes, das Joax oben zeigt, das Bild von "Kanonenkugeln" also Teilchen dar ?
Noe, das waere doch eine Gaussverteilung. Man erkennt beim Einzelspalt deutlich die Ueberschwinger der Sinus Kardinalis Funktion : |sin(kx)/kx|^2

Das ist das Betragsquadrat der Fouriertransformierten der Spaltfunktion und das zeigt, dass am Schirm eine Psi Welle angekommen ist. Die Anordnung stellt einen Fouriertransformator dar.
Oder noch einfacher :
A)
Ich betrachte einen Einzelspalt. Ich schicke ein Teilchen los. Und das ist eine Psi-Welle. Ich weiss, dass diese Welle durch den Spalt gelaufen ist, wenn es am Schirm detektiert wird. Gibt es irgendeine Veranlassung dazu, dass sich die Psi-Welle aufgrund meiner Information am Spalt ploetzlich in ein Teilchen verwandelt ?
Die Wellen, die Joax oben dargestellt hat, hat meine Information offenbar nicht gejuckt. Die sind wegen dem Spalt nicht dekohaeriert.

B)
Jetzt betrachte ich einen Doppelspalt und halte einen Spalt zu. Weiss die Welle dass das eigentlich ein Doppelspalt ist ? Man mutet der Quantenwelt vieles zu, aber ich meine nicht, dass die Welle das weiss. Dass ein boeser Beobachter mit fiesen Tricks indirekt ihre angeblich schizophrenen Verhaltensweisen studieren will. Sie verhaelt sich genauso wie beim Einzelspalt.

Beim Doppelspalt ergibt sich nun ein anderes Bild als beim Einzelspalt, weil die Anordnung aufgrund der Geometrie einen Foriertransformator darstellt.
EDIT :
Die Gitterfunktion wird mit dem Spalt gefaltet
http://home.arcor.de/richardon/2009/fourier2.gif
http://www.quanten.de/forum/showthread.php5?t=1102&page=35&highlight=Gitterfunktion

Anders sieht es aus, wenn ich beim Doppelspalt nun eine direkte Mess- Beobachtungsmethode verwende. Man bietet der Welle irgendetwas Heisses und vor allem Schweres an mit dem es wechselwirken kann. Dieses schwere Messgeraet gehoert zu unserer Realitaet, die dadurch ausgezeichnet ist, dass alle "Teilchen" fest miteinander verschraenkt sind und eine gemeinsame globale Entropie bilden, die die Richtung des Zeitpfeils unserer Realitaet festlegt. Nun verschraenkt sich ein Zustand der Welle mit dem Messgeraet (welcher scheint fuer uns zufaellig) und damit verschraenkt sich diese Moeglichkeit mit unserer Realitaet, die Moeglichkeit wird realisiert und wir interpretieren dies als Teilchen. Und ebenso interpretieren wir aus unserer Froschperspektive geraus, dass die Welle in diesem Moment scheinbar kollabiert. Sie ist aber mit einer endlichen Dekohaerenzzeit die von m und T abhaengt zuvor dekohaeriert. Die Moeglichkeiten bleiben weiterhin miteinander verschraenkt, die Welle bleibt weiter Welle, sie hat lediglich ihre Lage im Konfigurationsraum geaendert und steht nun orthogonal zu unserer Realitaet. Und die Moeglichkeit die unsere Realitaet schneidet interpretieren wir als Teilchen.
Das waere in etwas ein Beschreibung ueber die Dekohaerenz. Prof Zeh koennten sicherlich sogar eine richtige Beschreibung liefern :D

Und wenn das realisierte Teilchen ein Photon ist bin ich im Grunde auch etwas ratlos. Denn kaum hat es sich in unserer schnoeden Welt realisiert, dekohaeriert, ist es ja schon wieder daraus verschwunden, wenn es in unserer Realitaet etwas bewirken soll. Ich wuerde mir dies so erklaeren, dass das Photon tatsaechlich auf nimmerwidersehen verschwunden ist, wenn seine Wirkung irreversibel ist. Es sind ja zwei Parameter bei der Dekohaerenz. Masse und Temperatur. Wenn das Photon an einem thermodynamischen Umsatz, einer Entropieaenderung beteiligt ist, dann ist der Vorgang auf jeden Fall irreversibel. Bei gravitativen Effekten muss dem wohl nicht unbedingt so sein. (Mehrkoerperproblem?)
Da koennte es somit wieder als Psi Welle sich von unserer Realitaet "entschraenken".

Damit haette jedes "Photon" das nicht absorbiert wird Wellencharakter.

IMHO
Gruesse

Eyk van Bommel
09.10.11, 19:22
Oh doch Eyk!
Ähm - Oh nein?:D

Die Bombe ist ja sozusagen in Superposition! Also die Bombe ist zwischen „scharf“ und „nicht scharf“. Aber weder das eine noch das andere. Sie schwankt/wechselt zwischen „0 und 1“. Kommt das Photon auf dem Weg A gerade im richtigen Moment vorbei (Zwischenzustand 0), dann wird es nicht absorbiert. Im falschen Moment (Zwischenzustand 1) explodiert die Bombde (und die Katze ist tot ;)).

Also wir haben genau, dass was du suchst oder? 50% der Photonen werden absorbiert (auf dem Weg A).

Nun senden wir ein Photon über B! In dem Moment, in dem es auf dem Weg A auf die Bombe getroffen WÄRE, spürt das Photon den Zwischenzustand 0 oder Zwischenzustand 1.

Ich nenne es mal das durchfliegen einer virtuellen (sozusagen projizierten) Bombe auf dem Weg B.

WÄRE die Bombe auf 0 gestanden, dann verhält sich das Photon exakt wie sein „Zwillingsbruder über A“ es „fliegt durch die virtuelle Bombe“ und weist am Ende eine Interferenz auf. WÄRE (indem Moment die) Bombe auf „1“ gestanden, dann verhält es sich so, als ob man nun wissen würde, es hat den Weg B genommen. Keine Interferenz.

Wenn das Photon über den Weg B keine Interferenz aufweist, dann weis man, dass die Katze indem Moment als das Photon die „virtuelle Bombe“ durchflog tot war!

Gruß
EVB

@ richy
Mir ist das zu hoch. Gehts auch einfacher ?

Und dann kommt so was..:confused: ... also richy-Das soll einfacher sein:D

Aber wie siehst du das (oder Prof Zeh ;) )? Stelle in einen der beiden Öffnungen ein Teilchen in Superposition. In einem Zustand zwischen "photosensitiv" und "durchsichtig".

Da ist nichts
„Man bietet der Welle irgendetwas Heisses und vor allem Schweres an mit dem es wechselwirken kann.“
Das ist ganz Klein und halbdurchlässig.

Gruß 2
EVB

amc
09.10.11, 21:13
75% interferieren heißt in diesem Experiment, dass 100% von denen, die durchkommen/detektiert werden, im rechten Detektor landen. => Keins landet im oberen.Nein das heißt es nicht, nur dann, wenn die Wahrscheinlichkeiten der Wege identisch sind. Wenn nicht, dann wird es zu Phasenverschiebungen kommen!

Grüße, AMC

Diese Aussage muss ich zurückziehen. Ich weiß nicht wie das Verhalten wäre. Kann durchaus sein, dass eine unerschiedliche Wahrscheinlichkeit der Wege keinen Einfluss auf die Phasenverschiebung hat, somit tatsächlich auch dann alle ankommenden Photonen im rechten Detektor registriert werden.

Klar ist jedenfalls, wenn von den 75% Photonen 1/3 mit Teilchencharakter, also mit genau definiertem Weg, an dem zweiten halbdurchlässigen Spiegel ankommen, dann muss sich ein anderes Verhalten zeigen, als wenn alle der 75% mit Wellencharakter, also nicht genau definerten Weg, ankommen.

Grüße, AMC

amc
09.10.11, 21:32
Ähm - Oh nein?:D

Die Bombe ist ja sozusagen in Superposition! Also die Bombe ist zwischen „scharf“ und „nicht scharf“. Aber weder das eine noch das andere. Sie schwankt/wechselt zwischen „0 und 1“. Kommt das Photon auf dem Weg A gerade im richtigen Moment vorbei (Zwischenzustand 0), dann wird es nicht absorbiert. Im falschen Moment (Zwischenzustand 1) explodiert die Bombde (und die Katze ist tot ;)).

Aha, hier haben wir den Knackpunkt. Es ist nirgendwo davon die Rede, die Bombe sei in Superposition, ihr Zustand sei nicht definiert. Weder im Wiki-Eintrag noch im Video wird dies behauptet. Nein, es ist lediglich so, dass wir nicht wissen, ob es sich um eine scharfe Bombe mit Zünder handelt, oder um eine Bombe ohne Zünder.

Kommen nun alle Photonen im rechten Detektor an, dann wissen wir, die Bombe hat keinen Zünder (kein Objekt versperrt den Weg). Dass die Bombe einen Zünder hat können wir anhand zweier Ereignissen feststellen: Erstens - sie explodiert (mist, das wars dann;) ), oder zweitens - der obere Detektor schlägt an, was nur passieren kann, wenn ein Hindernis (der Zünder) den einen Weg versperrt. Wir haben also ohne direkt zu messen festgestellt, ob die Bombe einen Zünder hat, dies wäre mit klassischer Physik nicht möglich. Jedoch hätte die Bombe dabei auch in die Luft fliegen können ...

All dies hat aber nichts mit Superposition der Bombe zu tun, der Zustand der Bombe ist zu jederzeit definiert, und wir wollen ihn nur in Erfahrung bringen.

Grüße, AMC

P.S. Man könnte sich natürlich einen analogen Versuch überlegen, wo ein Superpositionszustand eines Objektes auf dem einen Weg eingebaut wird. Aber dann wird es IMHO so sein müssen, wie ich schon sagte, dass alle ankommenden Photonen Inteferenzverhalten zeigen. Kommt kein Photon an wissen wir, es hat mit dem Superpositionsobjekt reagiert.

Wenn du noch der Meinung bist, es handelt sich bei der Bombe um einen Superpositionszustand, könntest du mir dann zeigen wo dies behauptet wird?

richy
09.10.11, 22:10
Hi AMC
Ich verstehe einfach die Problematik nicht.
Klar ist jedenfalls, wenn von den 75% Photonen 1/3 mit Teilchencharakter ....
Wie Teilchencharakter und Wellencharakter ? Ist das ein Gemischtwarenladen ? :D
Bei Elektronen koennte ich mir das noch vorstellen. Einige sind dekohaeriert, damit realisiert und andere nicht. Aber wenn die Psi Welle eines Photons (und das ist keine EM Welle) dekohaeriert, dann ist es selber verschwunden oder abgestrahlt. Ins Nirvana oder wie bei Benjamins Glasbeispiel zurueck als Psi Welle. Gibt es Photonen in Wellenform und in Teilchenform ? Das dekohaerierte Photon kann auch in Form eines EM Wellenpaketes auftreten. Ein EM Wellepaket ist prakisch das Teilchen. Was wollen wir nun Photon nennen ? Ein EM Wellenpaket oder die Psi Welle ? Ich tendiere fuer das EM Wellenpaket. Und das kann auch abgestrahlt werden z.B, in Form von Waerme. Und diese wandelt sich nicht wieder in die selbe Psi Welle um. Der Vorgang ist nicht reversibel.

Basiert deine Bezeichnung "wellenartige"/"teilchenartige" Photonen auf diesem Wiki Zitat ?
http://de.wikipedia.org/wiki/Wechselwirkungsfreie_Quantenmessung#Elitzur-Vaidman-Schema
Beim Doppelspaltexperiment können die Lichtquanten entweder den einen oder den anderen Weg nehmen. Wenn man feststellen kann, welchen Weg ein Lichtquant nimmt, so entsteht kein Streifenmuster, sondern abwechselnd eine Beleuchtung von der einen und dann der anderen Seite.
Und die Beleuchtung von der einen oder anderen Seite beweist dann, dass aus den Photonen Psi-Wellen ploetzlich Licht-Kanonenkugeln wurden ? Auf was bezieht sich denn dieser Satz konkret ? Auf den Versuchsaufbau ?
ebenso :
Wenn jedoch das Messobjekt eingefügt wird, verhält sich das Photon als Teilchen und nimmt mit einer Wahrscheinlichkeit von 50 % entweder den unteren Weg oder den oberen Weg.
Aus welcher Messgroesse geht hervor, dass nun ein teilchenartiges Photon entsteht ?
Gut. Der blockierende Koerper soll so beschaffen sein, dass sich die Welle mit diesem verschraenkt, in der Form, dass sie tatsaechlich dekohaeriert. Dann wird das Quant doch absorbiert oder ?Und dann ist es doch wech.

Wenn ich die PSI Welle stets als solche betrachte erhalte ich dann nicht das selbe Resultat im Versuch ?
Und wie solle sich das "teilchenartige Photon" als Welle beweisen wenn es in dem Versuchsaufbau mit blockiertem Weg gar keine Moeglichkeit durch Interferenz dazu gibt ? Ausser dass es ab und zu auch mal den rechten Detektor trifft.

Eyk van Bommel
09.10.11, 22:17
Hallo amc,

unter dem "Wiki-Link: Wechselwirkungsfreie Quantenmessung"

findest du:
Mit der wechselwirkungsfreien Quantenmessung kann man feststellen, ob Schrödingers Katze noch lebt, ohne die Katze dabei zu töten

Das kannst du nicht mit einer Bombe, die nicht in Superposition ist.

Dass das nicht erwähnt wird? Meine Frau würde sagen: Didaktische Reduktion:D

Wozu sonst dieses Konstrukt? Das wäre nichts anderes wie Detektor "an" oder "aus".

Gruß
EVB

PS: Ich finde man findet hier auch einen Unterschied zu Prof Zeh.

Bei Zeh (da kenne ich mich aber nicht aus) wäre die Bombe danach in einem anderen Zustand? Zwischen "explodiert" und "durchsichtig ". Bei der klassischen Sichtweise (da kenne ich mich aber genauso wenig aus:D ) zwischen "photosensitiv" und "durchsichtig".

richy
09.10.11, 22:23
Um meine einfachen sicherlich ungenauen Vorstellungen mal allgemein zu formulieren :
Das Dekohaerenzprogramm ist doch inzwischen sicherlich von den meisten akzeptiert. Gerade auch von Anton Zeilinger. Ob es da in Details Viele Welten oder einen Konfigurationsrum (ebenso wie in der Bohmschen Mechanik) gibt kann man mal ganz beiseite lassen. Dann sollte Wiki diesen Versuch doch ueber Dekohaernz erklaeren und nicht immer wieder diese 1 Bit Weg Information in den Vordergrund stellen.
Und das Dekohaerenzprogramm besagt, dass die Dekohaerenzzeit, also wie lange dieses Psi Photon eine kohaerente Welle darstellt von 2 Parametern abhaengt. Der Temperatur und der Masse. Wo finde ich in der Wiki Versuchsbeschreibung diese beiden Parameter ?

amc
10.10.11, 00:26
Mit der wechselwirkungsfreien Quantenmessung kann man feststellen, ob Schrödingers Katze noch lebt, ohne die Katze dabei zu töten

Das ist lediglich eine plakative Aussage. Eyk, es handelt sich doch hier nicht bloß um ein reines Gedankenexperiment, sondern um ein tatsächlich durchführbares! Wie willst du denn makroskopische Objekte dieser Größe in einen Superpositionszustand bringen?

Oder meinst du, wenn du makroskopische Objekte wie eine Katze, eine Bombe, oder was auch immer, in eine schwarze Kiste steckst, in die wir nicht hineinsehen können, dass diese Dinge sich dann automatisch in Superposition befinden? Es wäre skurril, würdest du dies glauben ...

Es handelt sich hier um makroskopische, klassische Objekte, deren Zustand zu jeder Zeit definiert ist!


Wozu sonst dieses Konstrukt? Das wäre nichts anderes wie Detektor "an" oder "aus".

Du hast den Versuch lediglich noch nicht genau verstanden. Oder ich hab ein Riesenbrett vorm Kopp ... ich gehe aber von Ersterem aus :)

Gute Nacht, AMC

amc
10.10.11, 00:37
Hi AMC
Ich verstehe einfach die Problematik nicht.

Hast du dir den genannten Versuchsaufbau angeschaut und verstanden? Wechselwirkungsfreie Quantenmessung (http://de.wikipedia.org/wiki/Wechselwirkungsfreie_Quantenmessung) - hier im Video wird er auch beschrieben: Video (http://homepage.univie.ac.at/franz.embacher/dasDing/video.html), den Anfang kann man überspringen.

Eyk geht davon aus, der Zustand der Bombe (hat einen Zünder/hat keinen Zünder) befindet sich in Superposition.

Der zweite Diskussionspunkt ist, wie verhält es sich, wenn man ein Objekt, welches die Photonen auf Weg A statt zu 100% lediglich zu 50% blockiert. Werden die übrigen 75% Interferenzverhalten zeigen, oder nur 50%, wobei dann 25% kein Interferenzverhalten zeigen, weil ihr Weg (nämlich Weg B) definiert ist. Ich gehe von 75% Interferenzverhalten aus, weil ich nicht erkennen kann wodurch ihr Weg gemessen sein sollte ...

Deinen restlichen Text hab ich mir noch nicht genau angeschaut. Mach ich morgen.

Gut's Nächtle ...

richy
10.10.11, 03:39
Hi AMC

So abwegig ist EvB's Annahme doch gar nicht. C70 Fulerene koennen im
Ueberlagerungszustand sein oder Atome zwischen den Zustaenden Atom und Molekuel oszillieren. Ich hab eine ganz anderes Problem bei dem Versuch.
Hast du dir den genannten Versuchsaufbau angeschaut und verstanden?
Das Prinzip hab ich schon verstanden. Blos habe ich Probleme mit einem Detail:
Ob ich jetzt Dekohaerenz oder einen Wellenkollaps annehme. Wie aendert diese Psi Welle (ich glaube das nennt man quant) ihren Charakter wenn der Weg blockiert ist ?
Nehme ich zunaechst das Modell, dass ein angenommenes Photon den blockierten Weg nimmt. Ok dann dekohaeriert die ganze Welle mit diesem Zustand und das erzeugte Photon wechselwirkt irreversibel, wird absorbiert.
Was ist aber andernfalls ?
Aus deiner folgenden Beschreibung koennte man meinen, dass du annimmst, dass in dem Fall das quant eine Zustandsveraenderung von Wellen zu Teilchencharakter vornimmt :
Klar ist jedenfalls, wenn von den 75% Photonen 1/3 mit Teilchencharakter, also mit genau definiertem Weg, an dem zweiten halbdurchlässigen Spiegel ankommen, dann muss sich ein anderes Verhalten zeigen, als wenn alle der 75% mit Wellencharakter, also nicht genau definerten Weg, ankommen.


Im Versuch steht :
Das Anschlagen des rechten Detektors entspricht jetzt dem Ergebnis ohne Objekt im oberen Strahlengang und lässt somit keinen Rückschluss auf das Vorhandensein des Objektes zu.

Das Quant fliegt somit nicht stur geradeaus wie eine Kanonenkugel sondern biegt schon mal auch nach rechts ab.
Betrachten wir nur den ersten Halbspiegel und positionieren dahiner zwei Detektoren. Was sollte hier interferieren ? Man wird das selbe Verhalten wie beim zweiten Spiegel mit Blockade feststellen oder ?
Es ist einfach keine zweite Welle da zum interferieren.
Ansonsten. Yepp zu 50% eine Messung ohne Wechselwirkung.

Eyk van Bommel
10.10.11, 07:36
Wie willst du denn makroskopische Objekte dieser Größe in einen Superpositionszustand bringen?

Es muss ja kein einzelnes großes Objekt sein!! Man kann (wie Anfangs beschrieben) auch „n-Moleküle“ in Superposition nehmen. Wobei man die Konzentration solange erhöht bis 100 % der Photonen absorbiert würden, wenn alle den „Zustand 1“ besitzen.
Ich habe am Anfang auf mal ein Plasma als mögliches Gas erwähnt. Hauptsache „das Teil das im Weg steht“ ist „unscharf“. Du könnest auch Elektronen nehmen, wann und wo sie im Weg stehen, ist vergleichbar mit dem Superpositionszustand …

Gruß
EVB

amc
10.10.11, 07:47
So abwegig ist EvB's Annahme doch gar nicht. C70 Fulerene koennen im
Ueberlagerungszustand sein oder Atome zwischen den Zustaenden Atom und Molekuel oszillieren.

Ich hatte ja auch schon gesagt, man kann sich natürlich einen Versuch mit Objekten überlegen, welche sich im Superpositionszustand befinden, nur bei den genannten Quellen (Wiki, Video) geht es darum überhaupt nicht.

amc
10.10.11, 07:48
Es muss ja kein einzelnes großes Objekt sein!! Man kann (wie Anfangs beschrieben) auch „n-Moleküle“ in Superposition nehmen. Wobei man die Konzentration solange erhöht bis 100 % der Photonen absorbiert würden, wenn alle den „Zustand 1“ besitzen.
Ich habe am Anfang auf mal ein Plasma als mögliches Gas erwähnt. Hauptsache „das Teil das im Weg steht“ ist „unscharf“. Du könnest auch Elektronen nehmen, wann und wo sie im Weg stehen, ist vergleichbar mit dem Superpositionszustand …

Du kannst dir natürlich solche Experimente Ausdenken, aber wie ich eben schon zu Richy sagte, es werden hier jedenfalls solche Versuche nicht beschrieben.

Und Guten Morgen ... :)

P.S. Hier im halben Gedankenversuch, also der mit der Bombe, da handelt es sich um eine Bombe, welche in einer Fabrik produziert wurde, und man weiß, manche Bomben haben einen Produktionsfehler, sie haben nämlich keinen Zünder, und wir wollen nun, ohne die Bombe mit einem Photon zu beschießen herausfinden, ob es sich um eine Bombe mit oder ohne Zünder handelt. Ob die Bombe einen Zünder hat, steht dabei aber die ganze Zeit fest, und wir wissen es lediglich nicht. Ob sie einen Zünder hat entschied sich als sie produziert wurde in der Fabrik. Das ist ein die ganze Zeit ein definierter, objektiv feststehender Fakt. Nichts mit Superposition!!!!!!!!!!!

JoAx
10.10.11, 11:11
Ich will mich aber nicht auf Wortklauberei einlassen,


Ich mich auch nicht. Es bestand offenbar ein Klärungsbedarf (und wenn auch nur meiner seits). Hat mir jedenfalls geholfen. Danke.


Gruß, Johann

JoAx
10.10.11, 11:18
Hallo richy!


Stellt das Bild des Einzelspaltes, das Joax oben zeigt, das Bild von "Kanonenkugeln" also Teilchen dar ?
Noe, das waere doch eine Gaussverteilung. Man erkennt beim Einzelspalt deutlich die Ueberschwinger der Sinus Kardinalis Funktion : |sin(kx)/kx|^2


Doch. so würden sich auch "Kanonenkugeln" verhalten. Das meint zumindest Feynman, wenn ich das richtig verstehe. Der Abstand zwischen den Maximas wäre aber so gering (wegen der kleinen Wellenlänge), dass man nur die einhüllende zu Gesicht bekommt.


Gruß, Johann

JoAx
10.10.11, 11:32
Hallo AMC!


Klar ist jedenfalls, wenn von den 75% Photonen 1/3 mit Teilchencharakter, also mit genau definiertem Weg, an dem zweiten halbdurchlässigen Spiegel ankommen, dann muss sich ein anderes Verhalten zeigen, als wenn alle der 75% mit Wellencharakter, also nicht genau definerten Weg, ankommen.


Lass uns die Sache einfach machen. (Noch ein "Vereinfacher"! :mad: :D )

Nehmen wir das einfache DS-Experiment, und machen folgendes:

1. Wir schicken so viele Photone, dass am Schirm N Treffer verzeichnet werden. (Wahrscheinlichkeitsverteilung P12)
2. Wir machen den linken Spalt zu, und warten, bis am (selben) Schirm weitere N/2 Treffer ankommen. (Wahrscheinlichkeitsverteilung P1)
3. Wie 2., aber mit dem rechten Spalt. (Wahrscheinlichkeitsverteilung P2)

Welches Bild wird sich uns zeigen?

P = P12 + P1 + P2

Einverstanden?


Gruß, Johann

JoAx
10.10.11, 13:07
Hallo richy!

Wie aendert diese Psi Welle (ich glaube das nennt man quant) ihren Charakter wenn der Weg blockiert ist ?


Gar nicht. Ich denke, man könnte es sich so vorstellen (eselbrückenhaft):

Wenn das Quant die Kanone verlässt, dann hat es ein Einfachspalt verlassen. Seine Aufenthaltswahrscheinlichkeit wird also bereits von einer ψ Funktion beschrieben. Am 1. Halbspiegel entstehen 2 neue ψ Funktionen, die dann "ihren Weg gehen", und irgendwann am zweiten Halbspiegel ankommen. Dort "entsteht" eine neue Wahrscheinlichkeitsfunktion, die sich aus der Superposition der beiden ergibt.

P12 = |ψ1 + ψ2|^2

P - Aufenthaltswahrscheinlichkeit. Klassisch wäre zu erwarten:

P12_Klassisch = |ψ1|^2 + |ψ2|^2

Ist eine der Funktionen "auf dem Weg verschwunden" ψ = 0, wird klassische Aufenthaltswahrscheinlichkeit gleich der quantenmechanischen.


Gruß, Johann

JoAx
10.10.11, 13:22
Hi AMC!

Das ist ein die ganze Zeit ein definierter, objektiv feststehender Fakt. Nichts mit Superposition!!!!!!!!!!!

Du darfst manches echt nicht zu ernst nehmen. Sonst müsste man ja auch "Schrödingers Katze" ablehnen. Was Eyk macht, ist eine Art Rollentausch. Der Punkt ist, dass du nicht weist, welche Bombe sich gerade in der Apparatur befindet. Es kann sowohl eine mit, wie auch eine ohne Zünder sein. Wie beschreibt man diese Situation? - Mit einer Überlagerung/Superpositionen beider Möglichkeiten für nur eine Bombe.

imho


Gruß, Johann

amc
10.10.11, 14:23
Du darfst manches echt nicht zu ernst nehmen. Sonst müsste man ja auch "Schrödingers Katze" ablehnen. Was Eyk macht, ist eine Art Rollentausch. Der Punkt ist, dass du nicht weist, welche Bombe sich gerade in der Apparatur befindet. Es kann sowohl eine mit, wie auch eine ohne Zünder sein. Wie beschreibt man diese Situation? - Mit einer Überlagerung/Superpositionen beider Möglichkeiten für nur eine Bombe.

Hallo Johann,

nein. So kann man das nicht sagen. Das wird der Sache nicht gerecht. Es muss sogar schlicht falsch sein, es so zu formulieren. Der Zustand der Bombe (Zünder ja/nein) befindet sich definitiv nicht in Superposition. Es ist ein ganz entscheidender Unterschied, ob der Zustand eines Objektes tatsächlich nicht definiert ist (Superposition), oder wir ihn lediglich nicht wissen. Und weil dies ein ganz entscheidender Unterschied ist, hat dies auch einen wesentlichen Einfluss auf das Versuchsergebnis.

Stelle dir mal dies vor:

Person A entscheidet ob ein Objekt in unseren Versuchsapparat hineingelegt wird oder nicht. Wenn sie/er es hineinlegt, wird einer der Wege von ihm versperrt (analog Bombe mit Zünder). Person B weiß nun nicht, wie sich Person A entschieden hat, kann aber anhand des beschriebenen Versuchsaufbaus, unter Zuhilfenahme der quantenmechanischen Nichtlokalität, herausfinden, ob dieser Gegenstand hineingelegt wurde oder nicht, ohne ihn dabei in jedem Fall direkt messen zu müssen.

So - hier ist es doch nun nicht so, dass das Objekt für Person A einen definierten Zustand besitzt, und sich aber für Person B in Superposition befindet. Nein, es handelt sich hier um ein objektives Faktum, ob der Zustand definiert ist, oder nicht. Genau dies ist auch die, selbst in der QM einzuhaltene und gegebene Objektivität, welche, so wie ich es verstehe, Wolfgang Pauli mit seiner Aussage gemeint hat (ihr werdet euch sicher noch an die Diskussion erinnern? ;) )

Viele Grüße, AMC

amc
10.10.11, 14:30
Nehmen wir das einfache DS-Experiment, und machen folgendes:

1. Wir schicken so viele Photone, dass am Schirm N Treffer verzeichnet werden. (Wahrscheinlichkeitsverteilung P12)
2. Wir machen den linken Spalt zu, und warten, bis am (selben) Schirm weitere N/2 Treffer ankommen. (Wahrscheinlichkeitsverteilung P1)
3. Wie 2., aber mit dem rechten Spalt. (Wahrscheinlichkeitsverteilung P2)

Welches Bild wird sich uns zeigen?

P = P12 + P1 + P2

Einverstanden?

Ja, ich denke schon. Scheint mir eindeutig zu sein. Im ersten Schritt sind beide Spalte geöffnet (zwei mögliche Wege), also wird sich nach einiger Zeit ein Interferenzmuster (Wellenmuster, Streifenmuster) ergeben. Im zweiten Schritt werden nun jeweils einzeln nacheinander der rechte und der linke Spalt geöffnet. Hier wird dann jeweils, da nur jeweils eine Weg-Möglichkeit gegeben ist, ein einzelner Streifen auf der eher rechten Seite, und ein einzelner Streifen auf der eher linken Seite hinzukommen.

Hast du es so gemeint?

Grüße, AMC

Eyk van Bommel
10.10.11, 14:40
Hi amc,
wenn dem so ist wie du sagst (und ich glaube dir fast mittlerer weile!), dann ist die ganze Sache ja völlig unspektakulär!:confused:

Damit können wir also nicht automatisch auf einen Zustand schließen, wenn man dem Objekt/Teilchen keinen festen Wert zuordnen kann? Keine Messung der Katze im Sack – oder sowas.

Also sind wir wieder am Anfang!! Es gibt keine Experimente dazu:rolleyes:

Danke amc!:mad:

;)
Gruß
EVB

amc
10.10.11, 14:54
Klar ist jedenfalls, wenn von den 75% Photonen 1/3 mit Teilchencharakter, also mit genau definiertem Weg, an dem zweiten halbdurchlässigen Spiegel ankommen, dann muss sich ein anderes Verhalten zeigen, als wenn alle der 75% mit Wellencharakter, also nicht genau definerten Weg, ankommen.

Moin Richy,

hier erstmal ein wichtiger Hinweis: dieses Zitat von mir bezieht ich nicht exakt auf den im Wiki-Eintrag beschriebenen Versuchsaufbau. Dort (Wiki-Eintrag) wird nämlich der Fall behandelt, wenn sich ein Objekt auf dem einen Weg befindet, welches mit 100% Wahrscheinlichkeit die Photonen auf diesem Weg absorbiert, sofern das Objekt denn vorhanden ist, was man eben durch den Versuch herausbekommen kann, ohne direkt nachzusehen.

Mein Zitat bezieht sich auf den leicht abgewandelten Versuch, wenn sich nämlich ein Objekt auf dem einen Weg befindet, welches lediglich zu 50% Wahrscheinlichkeit die Photonen auf diesem Weg absorbiert.

Aus deiner folgenden Beschreibung koennte man meinen, dass du annimmst, dass in dem Fall das quant eine Zustandsveraenderung von Wellen zu Teilchencharakter vornimmt :

Zuerst mal: Mit Quant bezeichnet man, nach meinem Verständnis, vor allem einen tatsächlichen Energieübertrag. Und die Welle beschreibt lediglich die Wahrscheinlichkeit, mit welcher sich dieser Energieübertrag (Quant) (bei einer Messung) ergibt, und dadurch die Teilchen lokalisiert. Bei der Welle selbst von einem Quant zu sprechen halte ich für nicht korrekt, da wir bei der Welle, nach Lehrmeinung ;) , nicht von einer physikalischen Existenz sprechen können. Kann aber auch richtig sein, ich weiß nicht viel. Ich jedenfalls würde mit Quantenobjekt schon auch (wie du?) den ganzen Vorgang beschreiben, also Ausgangsereignis, Wellenfunktion und Ergebnis. Das alles verstehe ich inbegriffen, wenn ich sage, das Quantenobjekt verhält sich so und so ...

Ist ja aber auch an dieser Stelle egal. Darum wollen wir uns jetzt nicht weiter kümmern.

Aus meiner Aussage könnte man schließen, einmal fliegt ein Photon als Welle, und einmal als Teilchen durch den Versuchsapparat, ja, das stimmt, so meine ich es aber nicht! So formuliere ich nur um besser drüber reden zu können. Man kann immer nur Aussagen in dieser Form machen: Hier registrieren wir das Photon wurde losgeschickt, und dort registrieren wir es ist angekommen. Ob und wie es sich tatsächlich von A nach B bewegt, darüber kann man keine echten wissenschaftlichen Aussagen machen.

Das Anschlagen des rechten Detektors entspricht jetzt dem Ergebnis ohne Objekt im oberen Strahlengang und lässt somit keinen Rückschluss auf das Vorhandensein des Objektes zu.Das Quant fliegt somit nicht stur geradeaus wie eine Kanonenkugel sondern biegt schon mal auch nach rechts ab.

Auch eine Kanonenkugel kann natürlich abgelenkt werden. Nur mal so als Hinweis ... :)

Ja, hier verhält es sich dann wieder so, dass beide Detektoren anschlagen können, weil es nur eine Weg-Möglichkeit gab, nämlich der untere Weg. Am zweiten Strahlteiler kommt nun nur noch die Welle des einen Weges an (die des Unteren), und nicht mehr auch die des oberen Weges, wodurch sich die, nach dem zweiten Strahlteiler nach oben führende Welle (zum oberen Detektor) des unteren Weges, nicht mehr aufgrund der Phasenverschiebung mit der zuvor von links kommenden Welle (ohne Objekt) gegenseitig auslöschen kann.

Also gilt mit Objekt:
Weg A - Photon wird gestoppt: Wahrscheinlichkeit = 0,5
Weg B - Detektor oben: Wahrscheinlichkeit = 0,25
Weg B - Detektor rechts: Wahrscheinlichkeit = 0,25

Wobei sich, wenn der rechte Detektor einmal anschlägt, noch nicht sagen lässt, ob das Hindernis nun da war oder nicht. Weil sich dieses nicht unterscheiden lässt von dem Fall, wenn das Objekt nicht vorhanden ist.

Ohne Objekt:
Weg nicht definiert / Beide Wege möglich - Detektor rechts: Wahrscheinlichkeit = 1

Betrachten wir nur den ersten Halbspiegel und positionieren dahiner zwei Detektoren. Was sollte hier interferieren ? Man wird das selbe Verhalten wie beim zweiten Spiegel mit Blockade feststellen oder ?
Es ist einfach keine zweite Welle da zum interferieren.

Ja, es ist genau das selbe Verhalten wie am zweiten Strahlteiler, sofern das Quantenobjekt nur auf einem Wege zu diesem kommen konnte. Genau, ab hier gibt es dann zwei Wellen (mögliche Wege).

Ansonsten. Yepp zu 50% eine Messung ohne Wechselwirkung.

Wobei man eben, wie auch schon oben und in den Quellen (Wiki,Video) getan, anmerken muss, dass sich anhand eines Anschlages des rechten Detektors noch keine Aussage treffen lässt.

Viele Grüße, AMC

amc
10.10.11, 15:00
wenn dem so ist wie du sagst (und ich glaube dir fast mittlerer weile!), dann ist die ganze Sache ja völlig unspektakulär!:confused:

Hi Eyk,

wie ich schon sagte, im Grunde nichts Weltbewegendes. Jedoch so ganz unspektakulär ist das Ganze natürlich auch nicht. Gerade für Menschen, die sich mit QM nicht beschäftigen, ist dies ein wunderbares Beispiel, um die verrückte Nichtlokalität nahezu bringen.

Also sind wir wieder am Anfang!! Es gibt keine Experimente dazu

Ja, und da wirst du auch keine Möglichkeit finden. Einen tatsächlich undefinierten Zustand kann man nicht messen, ohne ihn zu messen. Nur einen bereits definierten Zustand, kann mit Hilfe der Nichtlokalität indirekt feststellen. Weil dieser bereits definierte Zustand, einen nichtlokalen Einfluss auf das zukünftige Verhalten meines Quantenobjektes hat.

Danke amc!

Gerne, irgendwann liege ich mal daneben und du hilfst mir auf die Sprünge.

Bis dann, AMC

JoAx
10.10.11, 15:51
Hi AMC!


Hast du es so gemeint?


Genau so. Und jetzt vermische diese 3 Experimente beliebig untereinander. Soll heißen - nach jedem Treffer auf dem Schirm wählst du (oder eine Maschine) zufällig einen anderen Aufbau. Ändert sich etwas am Ergebnis, wenn du 2N Treffer abgewartet hast? Doch wohl eher nicht.

Aber nichts anderes läuft ab, wenn man halbdurchlässiges "Medium" als "Spaltabschirmung" nimmt. Oder?


Gruß, Johann

richy
10.10.11, 17:17
Hier nochmals eine etwas genauere Erklaerung des Versuchs :
http://n.ethz.ch/student/jfink/handout.pdf
Eine Moeglichkeit dem Paradoxon, ohne Interaktion Information zu erhalten, zu entkommen waere die Viele-Welten Interpretation (MWI). Hier gibt es den Kollaps der Wellenfunktion nicht, denn alle moeglichen Zustaende sind real. Die drei verschiedenen Ergebnisse manifestieren sich in drei verschiedenen Welten. Nachdem alle drei Welten Teil der Physikalischen Realitaet sind kann man nicht davon sprechen dass keine Interaktion stattgefunden hat.

Ich wuerde dies etwas anders formulieren :
denn alle moeglichen Zustaende sind physikalisch ....
Nachdem alle drei Welten Teil der Physikalitaet sind ....

Das entspricht indirekt deiner Erklaerung.
Weil dieser bereits definierte Zustand, einen nichtlokalen Einfluss auf das zukünftige Verhalten meines Quantenobjektes hat.
Man hat sich im Grunde schon so sehr an die Nichtloalitaet gewoehnt, dass man sie einfach akzepiert.
Da besondere an dem Versuch duerfte gegenueber dem DS sein, dass hier die Phase fest vorgegeben ist und die Interferenz mit einem Ereignis verknuepft ist.

richy
10.10.11, 17:54
Doch. so würden sich auch "Kanonenkugeln" verhalten. Das meint zumindest Feynman, wenn ich das richtig verstehe. Der Abstand zwischen den Maximas wäre aber so gering (wegen der kleinen Wellenlänge), dass man nur die einhüllende zu Gesicht bekommt.
Den Begriff Kanonenkugeln habe ich verwendet, weil der oft in Beschreibungen benuetzt wird. Ich meine auch von Lesch. Du meinst auch Kanonenkugeln haben Wellencharakter. Ja, das haben sie. Ich wollte nur darauf hinweisen, dass dieser Wellencharakter eben auch schon beim Einzelspalt messbar ist. Und dieser fuehrt dazu, dass das Detektorbild die Fouriertransformierte der Spaltgeometrie ist.
http://www.pi5.uni-stuttgart.de/lectures/88/Versuch_Beugung_Anleitung.pdf
Vergleicht man Gleichung (9) mit der Fouriertransformierten .... einer Funktion f(x) mit der Wellenzahl k = 2x R, so liefert dies ein erstaunliches Ergebnis: Die Intensitaetsverteilung der Fraunhoferbeugung im Fernfeld ist also gerade die Fouriertransformierte der Spaltfunktion p(x).

Und natuerlich haengt das mit der Unschaerferelation direkt zusammen. Und wenn man konsequent dieses unselige komplementaerargument anwenden wuerde, dann muesste man entweder der Spaltgeometrie oder der Intensitaetsverteilung eine Realiatet sogar Phyikalitaet absprechen. Ist die Spaltgeometrie nur gedacht ? Das wuerde niemand behaupten. Also ist die Intensitaetsverteilung nur gedacht ? Das waere die Annahme der KI. Und so koennte man schon argumentiern. Weil der Detektor das gesamte Bild aus einzelnen Ereignissen zusammensetzt. Das Bild wird nicht auf einen Schlag realisiert. Je nach Technik stellt der Detektorschirm einen Kollektor dar. Der Ereignisse verschiedener Zeitpunkte (Realitaeten) sammelt. (Nur die Gegenwart scheint uns real)
Dem kann man nun entgegenwirken indem man 10 000 identische DS Versuche aufbaut und alle Ergebnisse ueberlagert. Das ginge dann in Richtung VWI,BM. Oder man sagt einfach. Ok das Gesamtbild ist in dem Sinne nur gedacht. (Ein Teilnehmer im Forum hat sogar bezweifelt, dass dies eine Messung des Wellencharakters darstellt). Aber es ist doch einsichtig, dass jeder Detektorpunkt fuer sich physikalisch ist. Ok meinetwegen ist das Gesamtbild, der Wellencharakter irreal, aber jeder einzelne Punkt ist physikalisch und damit das Gesamtbild und damt der Wellencharakter physikalisch. IMHO

Gruesse

Eyk van Bommel
10.10.11, 20:07
Einen tatsächlich undefinierten Zustand kann man nicht messen, ohne ihn zu messen.
Wenn ich das bisschen über die VWI richtig interpretiert habe, geht man doch dort davon aus, dass beide Zustände real sind nur in unterschiedlichen Realitäten. Bei einer Messung wird eine der beiden Zustände nur in unsere Realität geholt –oder?

Gruß
EVB

amc
10.10.11, 21:38
Wenn ich das bisschen über die VWI richtig interpretiert habe, geht man doch dort davon aus, dass beide Zustände real sind nur in unterschiedlichen Realitäten. Bei einer Messung wird eine der beiden Zustände nur in unsere Realität geholt –oder?

Nur um Missverständnissen vozubeugen: Meine Aussagen bezogen sich keinesfalls auf die VWI. Ich halte nicht sehr viel davon, zumindest glaube ich, wir benötigen alle Quantenphänomene um hier, unsere eine Welt, umfänglich erklären zu können. Ich bin aber (möglicherweise nur aus meiner Unkenntnis heraus) nicht abgeneigt, der Wahrscheinlichkeitswelle eine gewisse Physikalität zuzusprechen.

Ob in der VWI auch die Zustände vor den eigentlichen Ereignissen als real angesehen werden, oder ob sich lediglich alle möglichen Ereignisse in den verschiedenen Welten realisieren, dazu kann Richy bestimmt was sagen.

Grüße, AMC

richy
10.10.11, 22:55
Es gibt nur eine physikalische Realitaet fuer uns. David Deutsch hat diese Viele Welten auch etwas zu blumig ausgeschmueckt. Und Everetts Interpretation hies anfangs auch ganz anders. Mit diesen anderen Welten teilen wir die Raumzeitbuehne des Universums. Aber sie sind in meiner Vorstellung fuer uns nicht real. Lediglich physikalisch. Im Grunde ist der Unterschied zu Zeilinger nur noch dieses Detail. Und er erwaehnt halt seine informativen viele Welten nicht.
Feynman hat Everett uebrigends vor allem abgelehnt, weil es kein Dekohaerenzprogramm gab und damit musste man unendlich viele Welten annehmen. Die Dekohaerenzzeit fuehrt jedoch zu einer begrenzten Anzahl. Vielleicht 100! oder 2^100! Welten. Jedenfalls nicht wenige :-)
Aber das ist bei dem Versuch wohl eher ein Randthema. Messen ohne das Objekt zu messen. Velleicht koennte man in dem Versuch tatsaechlich noch mit Bertelmanns Socken argumentieren.


Gruesse

amc
11.10.11, 17:25
Genau so. Und jetzt vermische diese 3 Experimente beliebig untereinander. Soll heißen - nach jedem Treffer auf dem Schirm wählst du (oder eine Maschine) zufällig einen anderen Aufbau. Ändert sich etwas am Ergebnis, wenn du 2N Treffer abgewartet hast? Doch wohl eher nicht.

Hallo Johann,

sehe ich auch so. Du meinst also, es wird immer vor einem Photonenschuss (per Zufall) entschieden (jeweils Wahrscheinlichkeit 0,33): entweder nur der linke Spalt offen, nur der rechte Spalt offen, oder beide offen? Ja, hier dürfte sich, meine ich, kein anderes Ergebnis zeigen, als zuvor.

Aber nichts anderes läuft ab, wenn man halbdurchlässiges "Medium" als "Spaltabschirmung" nimmt. Oder?

Du meinst, vor beide Spalte wird z.B. ein halbdurchlässiger Spiegel gestellt? Ich denke, hier dürfte sich nun wieder ausschließlich ein Interferenzmuster ergeben, allerdings kommen nur 50% der Photonen durch. Für die, die durchkommen, dürfte der Weg aber nicht definiert sein, und sich daher auschließlich das Interferenzmuster zeigen. So stelle ich es mir vor.

Viele Grüße, AMC

Eyk van Bommel
11.10.11, 17:31
Velleicht koennte man in dem Versuch tatsaechlich noch mit Bertelmanns Socken argumentieren.
Ob, die Socken da helfen?
Aber das ist bei dem Versuch wohl eher ein Randthema. Messen ohne das Objekt zu messen.
Findest du?

Wie sehen die Ergebnisse in der V (knapp 100 ;)) WI aus?

Also das Photon schlägt den Weg A ein. Kommt an der Bombe vorbei (Zustand 0) = keine Wechselwirkung = Interferenz

Also das Photon schlägt den Weg A ein. Kommt an der Bombe vorbei (Zustand 1) = Wechselwirkung = Explosion.

Also das Photon schlägt den Weg B ein. Keine Bombe im Weg = keine Wechselwirkung = Interferenz

Hier (auf dem Weg B) werden 100% der Photonen also unabhängig von A ankommen? In der VWI ist es also egal, ob der Weg versperrt ist?

Gruß
EVB

PS: Ich kann die Lorentz-Traffo über die SRT (derzeit beste Modell) oder über den Lorentz-Äther (falsches Modell) nachvollziehen. Das würde ich nur gerne auch für die QM. Ich komme z.B. häufiger über den Lorentz-Äther einfacher zum Ziel - vielleicht auch bei der VWI.:)

JoAx
14.10.11, 14:49
Hallo AMC!

Ich denke, hier dürfte sich nun wieder ausschließlich ein Interferenzmuster ergeben, allerdings kommen nur 50% der Photonen durch.


Ja, ich denke, dass du Recht hast.
Problem erkannt, Problem gebannt.
Irgendwie habe ich ad hoc angenommen/postuliert, dass die Welle, die die Aufenthaltswahrscheinlichkeit beschreibt, entweder "durchkommt" oder "reflektiert" wird. Das muss aber bei den Funktionen quantitativ durch etwas ausgedrückt werden - Amplitude.
Da habe ich so etwas im Excel erstellt:

sinc(x), A1=1/3 und A2=2/3, Spalten bei x=1 und x=-1
http://www7.pic-upload.de/14.10.11/1tm8snaegls1.jpg

Dazu die Interferenz:
http://www7.pic-upload.de/14.10.11/xxxllqrvupuo.jpg

Ich denke, das stimmt so.
Danke! Hat mir was gebracht. :)


Gruß, Johann

amc
14.10.11, 15:01
Danke! Hat mir was gebracht. :)

Mir auch. Also, auch von mir ein Danke ...

Grüße, AMC